11-PrepTest 11 Expls

97 downloads 3131 Views 261KB Size Report
LSAT. RELEASED TEST XI. EXPLAINED. A Guide to the June, 1994 LSAT. KAPLAN ...... It's very similar to the third game of PrepTest X (the June 1994 exam ), “The. Word Game” — in ...... prevalence of women's medical practice” (lines 58- 59).
KAPLAN LSAT PREP

LSAT RELEASED TEST XI EXPLAINED A Guide to the June, 1994 LSAT

KAPLAN The answer to the test question.

1994 Stanley H. Kaplan Educational Center Ltd All rights reserved. No part of this book may be reproduced in any form, by photostat, microfilm, xerography or any other means, or incorporated into any information retrieval system, electronic or mechanical, without the written permission of Stanley H. Kaplan Educational Center Ltd.

SECTION I: LOGIC GAMES

© K A PL A N

1

LSAT PREP ________________________________________________________________ LSAT Test XI Explained: Section I

GAME 1 — Counselors and Activities (Q. 1–6) The Action: Assigning each of eight counselors to supervise exactly one of three activities at a camp. Nothing fancy here; simply a run-of-the-mill grouping game of distribution. The Key Issues that the game will test involve typical grouping concerns: 1) Which activity does each counselor supervise? 2) Which counselors can or must supervise the same activity? 3) Which counselors cannot supervise the same activity? The Initial Setup: In such straightforward grouping games of distribution, it’s usually helpful to create a sketch containing the groups that the entities are to be broken into, in this case, the 3 activities:

FGHJKLNO

Swim

Ten

V’ball

With nothing yet to fill in, it’s time to move right to the rules. The Rules: No surprises here; we get exactly what we’d expect from a grouping game of distribution’s rules — namely, statements that speak to the key issues stated above: 1) A number rule: Each activity must get at least two, but no more than three, counselors. Don’t make the mistake of just writing this down: Number rules should be taken to the max. The specified minimum of two counselors in each activity takes care of six of the eight counselors. And what about the other two? Can they be assigned together? No, for that would result in a group of four. Thus the remaining two counselors must split up. And that, in turn, tells us that the number breakdown among the sports has to be 3/3/2 at all times. 2) The most concrete rule of the bunch — put it right into your sketch, H under the Swimming column. 3) “No K or O” above “Tennis” in your sketch should be a sufficient reminder of this rule. You might also take the negative statement to a positive, and note for yourself that K and O must supervise Swimming or V’ball. 4) Make sure you interpret this one correctly: J can’t supervise the same activity as K, AND J can’t supervise the same activity as N. We can’t, however, infer from this rule that K and N supervise the same activity — their respective relationships to Joan are discussed, not their relationship to each other. 5) Finally, we get an if-then statement for Rule 5: If G supervises swimming, N and O go into the volleyball group. Underneath your shorthand for this, write in the contrapositive: If EITHER N or O is NOT supervising volleyball, then G is NOT supervising swimming. 2

© K A PL A N

LSAT PREP ________________________________________________________________ LSAT Test XI Explained: Section I

Key Deductions: Most of the time key deductions come from combining two or more rules (Step 4 of the Kaplan 5-Step Approach). Here, nothing new becomes readily evident from combining the rules, and a potent strategy emerges from that fact (see the first “Big Picture” point, below). The Final Visualization: Here’s what your page might look like as you head into the questions:

3/3/2 No K, no O Swim Ten Never JK Never JN G swim ➔ N, O v’ball N or O not v’ball ➔ G not swim

V’ball

The Big Picture: • When you can’t combine the rules and make any new deductions in a game, that usually means that several questions will be answerable based on one or two rules taken by themselves. In other words, the key to a question will often involve simply finding the relevant rule or rules that pertain to the specific question situation. This is in contrast to the tactic of using key deductions to jump start each solution, which is the method of choice when such deductions exist. We’ll see how the former method plays out in Q. 1-3 especially. • When the testmakers offer information about numbers, they usually give you just the bare bones and leave it for you to translate it into something useful. If you just wrote down “at least 2, no more than 3” for Rule 1, you’re at a real disadvantage compared to the test-taker who spent a few extra seconds thinking through what the rule means in a concrete way. You’re much better off if you work out the 3/3/2 breakdown. So: Whenever a rule deals with numbers, take the time to think through the concrete possibilities. • Always invoke the contrapositive when given if-then rules. If one of the clauses of the original statement is a compound clause (such as the second half of Rule 5), then the contrapositive stipulates that if either or both of the situations are not met (that is, if Nathan is not supervising volleyball or if Olga is not supervising volleyball or if neither of them is supervising volleyball), then the original situation cannot be true (George cannot be supervising swimming). • Take negative statements to the positive. When a statement tells you what cannot be true, carry it further and figure out what can or must be true. The LSAT will reward you for it.

© K A PL A N

3

LSAT PREP ________________________________________________________________ LSAT Test XI Explained: Section I

The Questions: 1. (A) (C) violates Rule 1 (each activity requires at least two counselors). Rule 2 is accommodated in each choice, but Rule 3 is violated in (E), by having Kathy supervise tennis. (D) contradicts Rule 4 by placing J and K together, and (B) is in violation of Rule 5, which states that Olga should be in the volleyball group when George supervises swimming. • When facing an “acceptability” question, simply check the choices against each rule, eliminating the ones that violate the rules until you’re left with only one choice. If you do your work carefully, there’s no need to check the remaining choice; just circle it and move on. 2. (E) We’re given no new information, and are simply asked which pair can supervise swimming (along with Henry, of course). Rule 5 makes both (A) and (B) impossible; with G in the swimming group, both N and O must supervise volleyball. Similarly, both (C) and (D) are killed by Rule 4: J/K and J/N are forbidden at all times. As simple as that. • Confidence is key. When you’ve absolutely eliminated all but one choice, have the confidence to circle the remaining choice and move on. Don’t waste time in this one “testing out” whether J and O can join H in swimming, and worse yet, filling in the other groups just to really make sure it’s possible. This is unnecessary!! The best Logic Gamesters know how to save time, and so they have that additional time when they really need it. 3. (B) Once again, the rules allow us to quickly eliminate all of the wrong choices without a whole lot of thought. (Remember the Big Picture point above, about answering questions in games devoid of major deductions? If not, re-read it now.) Certainly no pair containing K or O can supervise tennis, according to Rule 3, which kills (A) and (E). (C) is out because Henry must supervise swimming (Rule 2) and (D) is no good because J and N can’t supervise the same activity (Rule 4). • Don’t assume that every question requires lots of drawing on your part. These first three, at least, can be answered quickly either by having a firm grasp of the rules in your head and/or by taking a quick glance at your master-sketch shorthand on the page.

4

© K A PL A N

LSAT PREP ________________________________________________________________ LSAT Test XI Explained: Section I

4. (D) Our first hypothetical piece of information: G and K join H in the swimming group. According to Rule 5, that forces N and O to supervise volleyball, and J, who can’t be paired with N or K, must supervise tennis. That leaves F and L; they both could go to tennis, or one could join N and O in volleyball, while the other would join J in tennis (3/3/2, remember). (D), therefore, could be true: Lewis could supervise volleyball. (A) Swimming is complete with H, G, and K; Fran must supervise tennis or volleyball. (B) Henry always supervises swimming. (C), (E) No; in this case J supervises tennis, and Nathan supervises volleyball. • When you’ve gone as far as the chain of deductions will allow, attempt to complete the picture by asking yourself the following two questions: Who’s left, and where could they go? F and L are sort of “free agents” here in that no rules are attached to either. Be aware that, for that reason, these two counselors will often be the ones left over to fill in the spaces. 5. (C) F and L join H in swimming. That group is now complete, leaving the other counselors to fill in the tennis and volleyball groups. The relevant rules at this point are Rules 3 and 4: O and K can’t supervise tennis, so both’ll have to supervise volleyball. Rule 4 stipulates that J can’t be with K or N, which means that J must supervise tennis, while N joins K and O in volleyball. The remaining counselor, G, must join J to fill out the tennis roster. Every counselor is placed, and according to this assignment, only (C) must be true. • If you get stuck, ask yourself: “Which rule or rules would have the most impact on the situation as it now stands?” The best Logic Gamesters have a knack for focusing on the right rules at the right time.

© K A PL A N

5

LSAT PREP ________________________________________________________________ LSAT Test XI Explained: Section I

6. (B) With any luck, you saw the quick intuitive way to solve this question. J and O must be together, but J can’t be with N, and there’s one particular scenario that teams O with N, a scenario that would cause a problem for J. We’re referring to Rule 5. If G supervises swimming, then N and O supervise volleyball, and J would have to join them according to the question stem. But Rule 4 says that J and N can’t be in the same group. Therefore, G can’t supervise swimming, and that leads directly to choice (B). It’s o.k. if you didn’t see this line of reasoning right off the bat. The other approach is simply to test out how J and O can be in the same group. O can’t supervise tennis, so J/O must supervise either swimming or volleyball. Let’s say J and O join H in swimming. K, who can’t supervise tennis, would supervise volleyball, so choice (C) is possible. F, G, L, and N are then left to fill in the tennis and volleyball groups, in any old way, so choices (D) and (E) are possible. However, if J and O supervise volleyball, K would have to supervise swimming (Rules 3 and 4), and F could be in that group as well, as long as N supervises tennis. (A) could therefore be true. The only choice left, the one that’s impossible, is (B). • If you keep on the lookout for creative ways to utilize the rules, you’ll sometimes stumble across intuitive approaches that will enable you to answer the more involved questions quickly. • As a last resort in a “cannot be true” question, you always have the option to get the answer by process of elimination. That is, after crossing off every choice that could or must be true, you will be left with the credited choice.

6

© K A PL A N

LSAT PREP ________________________________________________________________ LSAT Test XI Explained: Section I

GAME 2 — Firefighters’ Schedules (Q. 7–11) The Action: No question about it, the task here is sequencing; specifically, you’ll have to place firefighters in order across a five-day work week. The only slight aberration is that we’re not told that each firefighter must actually work during the week, only that exactly one firefighter works each day. This leaves open the possibility that in some orderings, not every firefighter will be listed. With that in mind, the Key Issues for this one are: 1) Who works on which days? 2) Who works before and after whom? 3) How many shifts (including zero) does each firefighter work? The Initial Setup: We have to dip into the rules to find the major piece of information that will help us visualize and map out the game. Rule 1 tells us two things: that none of the firefighters work on the weekend, and that exactly one firefighter works on each of the other days. From this we can infer that exactly one firefighter works on each day from Monday to Friday. You’ll be hard pressed to find a simpler Logic Games scenario to visualize than this. How might you work out this situation on paper if this were a real-life task? Probably no differently than this:

FGHI

J

M

T

W

Th

F

___ ___ ___ ___ ___ The Rules: 2) Two is the maximum number of days any firefighter can work in a week. You may have wanted to write “2 MAX” or “NO ONE 3 TIMES” to remind you of this (or just circled the rule to remember it). Notice that we’re still not explicitly told that any particular firefighter has to work at all during the week. This rule merely imposes a limit on the number of days any one of them can work. 3) Fairly straightforward; Fuentes, for example, can’t work on Monday and Tuesday, or on Thursday and Friday. Any firefighter working on consecutive days would violate this rule. A jotted-down reminder “NO ONE TWICE IN A ROW” might help. 4) If Fuentes never works later in the week than Jackson, then Fuentes must always work earlier in the week than Jackson. Of course, bear in mind that this rule is only in effect when both of them work during the week; as we’ve seen, it’s possible that one or both of them may not be scheduled to work. You might have chosen to shorthand this as: “IF F and J work, then F . . . J,” or (looking at it a different way) simply “NEVER J . . . F.” The trick will be working out this rule when either F or J (or both) work two days. But we’re getting ahead of ourselves. © K A PL A N

7

LSAT PREP ________________________________________________________________ LSAT Test XI Explained: Section I

5) Another outcome if a particular firefighter is chosen to work this week. “If H, then HG” is one way to take care of this rule, as long as you know at a glance that “HG” means that those two firefighters work on consecutive days in that order. Make sure you don’t make the mistake that some test-takers did: If G works, H may or may not work, we can’t tell; this rule only works in one direction. And obviously if Graber doesn’t work during the week, then Howell can’t work either (this is a form of the rule’s contrapositive). Key Deductions: One way to know whether a game will be ripe for making deductions is to see whether the rules contain common entities. Here, the only rules that mention specific entities are Rules 4 and 5, but they have no firefighters in common. It turns out that, similar to the previous game, this game contains no earth-shattering deductions lurking in the shadows. Sure, we can deduce from Rule 5 that H can’t work on Friday, because there would be no way to get G on the following day. But this is hardly the kind of information that blows a game wide open. And we can deduce that at least three, and as many as five, firefighters will work (because there are five assignments, and no one works more than two). But that, too, doesn’t get us too much further along. So just as in Game #1, once again we’ll have to narrow down which rules are relevant to each particular question, based on the “action” suggested by the stem. When you let the specifics of a question lead you to the exact rule that applies, your work becomes much easier. The Final Visualization: Here’s the information at our disposal as we move on to the questions:

No one twice in a row Never J … F If H ➔ HG

FGHI

No one 3x

J

M

T

W

Th

No H

F

___ ___ ___ ___ ___

The Big Picture: • Make sure that you don’t read things into the game that aren’t there. Many sequencing games require that every entity appear in the ordering, but they always specify when this is the case. For example, in this game, the testmakers would say “every firefighter works at least once during the week,” or something to that effect. In the absence of such a statement, we have to infer that it’s okay for a firefighter to be left out of the weekly rotation. Missing that inference — as some students discovered, to their sorrow — gets you into big trouble. • Don’t be intimidated or get flustered by the roundabout nature of the wording of certain rules, such as Rule 1. Often they actually translate into very simple and manageable pieces of information.

8

© K A PL A N

LSAT PREP ________________________________________________________________ LSAT Test XI Explained: Section I

• Use the subtle clues that the testmakers provide. If you hadn’t yet figured out that each firefighter isn’t required to work at least once during the week, you should have been tipped off to this by the wording of Rule 5: “If Howell works . . .” certainly implies that Howell need not work at all. The Questions: 7. (B) Looking for an impossible schedule — an “unacceptability” — is much the same process as the standard acceptability question: Simply scan the choices against each rule, and the first rule to be violated will reveal the answer. As it happens, Rule 4 leads us to (B): Fuentes’s second shift comes later in the week than Jackson’s shift, which is a no-no. • In a question like this, you could read through each choice and check it against each rule, instead of the method we employed above. However, the main advantage of our method is that it allows you to scan the choices, with one rule in mind at a time. Ultimately, this approach is faster and generally more reliable. 8. (D) In scanning the list of schedules to find choices that violate the rules, we quickly can eliminate (B), (C), and (E). (B) and (C) are actually in the unique position of violating both Rule 3 (the non-consecutive rule) and the hypothetical in the question stem, since (B) has Howell and (C) has Iman working on consecutive days. (E) goes against Rule 4, and (A) violates the question stem, as Howell is granted only one day off between his two working days. (D) remains as the only choice that satisfies both the original rules and the question stem. • When the question is about acceptability, always start by scanning the choices for rule violators. • If you’re unsure of the correct way to interpret a stem hypothetical (or a rule), take a second to come up with an example that will help clarify its meaning. For example, what does “at least two consecutive days off” mean? It means that F, H, G, F, J is okay, but F, I, F, H, G is not.

© K A PL A N

9

LSAT PREP ________________________________________________________________ LSAT Test XI Explained: Section I

9. (B) The inclusion of Fuentes and Jackson in the question stem practically screams for you to focus on Rule 4. We’re looking for something that can’t be true, and we know from Rule 4 that F must work earlier in the week than J. This tells us that J can’t work on Monday, and that F can’t work on Friday. The only choice that contains one of these statements is (B), which puts Jackson on Monday. You don’t even have to read the rest of the choice. No matter how you slice it, Jackson working on Monday results in Fuentes working later in the week, which violates Rule 4. • In choices (like these) that contain two conditions, if one part of the choice is impossible, then the entire choice is wrong even if the second part is valid. Logic Games are based on objective rules, conditions, and situations, so there’s no such thing as a partly valid statement. • Even when you can only make what seem to be minor deductions, scan the choices to see if those points appear in any of the choices. Only as a last resort, if there’s literally nothing to go on, should you methodically “try out” answer choices. 10. (B) With J on Thursday, the only way F can work twice is if F works Monday and Wednesday. Why? Because F must always work before J (Rule 4), but cannot work on two consecutive days (Rule 3). This kills choice (A). (B) is the only choice that works: G can work Tuesday, and either G or I can then fill out the schedule by working Friday. As for (C), H can’t work on Tuesday because then G would have to work the next day (Rule 5), but F is the firefighter working on Wednesday. And that’s the reason (D) and (E) are wrong — Wednesday’s already taken. • The questions don’t exist in a vacuum: The work you put into earlier questions will often help you answer later ones. We just got finished with the previous question involving Fuentes and Jackson, so by now their situation should be engraved in your mind. • When you’re given two pieces of hypothetical information, work with the more concrete piece first.

10

© K A PL A N

LSAT PREP ________________________________________________________________ LSAT Test XI Explained: Section I

11. (D) If G doesn’t work during the week, then H can’t work, says Rule 5. That leaves F, I, and J to fill in the schedule. Since there are a number of ways to schedule these three firefighters, there’s not much more work we can do before seeing what the choices have to offer. (A) Must F work exactly one day during the week? No, F can work twice — F I F J I. (B) F can also work only once, killing this choice: I F J I J. Incidentally, this acceptable schedule allows us to cross off (E) as well. (C) From both of the preceding examples, we see that this choice need not be true — Iman certainly can work twice during the week. (D) And that leaves (D): No matter how you try, there’s no way to disprove the validity of this choice. Trying to place I only once in the schedule invariably leads to violating either Rule 3 or Rule 4. Since we can’t come up with a contrary example (not to mention the fact that we’ve crossed off everything else), choice (D) must be right. • When you have very little to go on in a question, quickly scan the answer choices to see what the question is getting at. Here, the answer choices tell you that we’re dealing with a numbers issue, not necessarily a placement issue. Knowing this should align your thinking and therefore help you proceed toward the answer. • When you’re faced with a “must be true” question and have little concrete information to back you up, one viable method is to eliminate choices by producing examples that contradict them. For example, for choice (A), we tried to create a schedule where Fuentes doesn’t work exactly one day during the week, and since we saw that he can work two days, we knew that this choice need not be true. Kaplan calls this method of eliminating choices “proving the exception.” When you come to a choice where no exception is possible, you’ve found the correct answer. (Ideally, you hope to follow a chain of deductions directly to the correct choice. However, when that’s simply not an option, as in this case, the “proving the exception” method is your next best bet.)

© K A PL A N

11

LSAT PREP ________________________________________________________________ LSAT Test XI Explained: Section I

GAME 3 — The Housing Committee (Q. 12–19) The Action: A grouping game of selection: We have to choose five representatives from a group of nine people, with one of the representatives to serve as the chairperson. A fairly standard wrinkle in this type of game is included: The nine eligible people are themselves broken up into two groups; tenants and homeowners. The Key Issues that will appear over and over in the course of the game are: 1) The standard grouping game of selection issue: who’s in, who’s out. 2) Which people can serve together on the committee? 2) Which people can’t serve together on the committee? 3) Who will serve as chairperson for the committee? The Initial Setup: There’s nothing real complicated here, so we have yet another simple setup on our hands:

T FGJKM

h pqrs

Pick 5: ___ ___ ___ ___ ___ The Rules: 1) This one should sound familiar, since it’s very similar to Rule 1 of the first game of the section. And just like with that one, we can infer from this rule that there must be exactly three reps chosen from one of the groups (tenants/homeowners) and two from the other. 2) Simple enough: The chairperson must come from the group that supplies exactly two of the representatives. Tough to shorthand. Put a box around it on the page. The next four rules are very standard grouping rules — they dictate which people can, must, or cannot be selected together. In other words, these rules go a long way toward defining the key issues above. 3) If F, then Q. Remember to think through the contrapositive: If NO Q, then NO F. 4) The same format as the previous rule: If G, then K. Contrapositive: If NO K, then NO G. 5) J and M are inseparable: If we select one, we have to take the other. Another way to encode it in your memory: We must take either both J and M, or neither.

12

© K A PL A N

LSAT PREP ________________________________________________________________ LSAT Test XI Explained: Section I

6) Self explanatory: NEVER MP. Key Deductions: Unlike the previous two games, there are a few things to deduce up front here. Since M is a common entity to both Rules 5 and 6, we should look to combine those. And in fact, we can deduce from these two rules that neither J nor M can be selected with P. No great shakes, but it’s a little something extra we weren’t explicitly told. But there’s another, more subtle deduction possible: If the J/M pair is selected, then G can’t be. Do you see why? It’s because selecting G would force us to select K, which along with J and M would give us four tenants. That would leave room for only one homeowner, which violates the first rule. The Final Visualization: Here’s what your page might look like as you head into the questions:

T FGJKM h pqrs ___ ___ ___ ___ ___ 3/2

If F ➔ q If q ➔ F If G ➔ K If K ➔ G JM or JM Never Mp

The Big Picture: • A game containing eight questions is an opportunity to get more bang for your buck (or more appropriately, more points for your time). Preview the section, and if you find a game with more than the average six questions, look to handle that one early on. • When entities are broken up into two distinct groups, you may wish to visually distinguish them on your page by using capital letters for one group and lower-case letters for the other. • Remember: When rules (like Rule 1) are about numbers, work out the arithmetic. • In grouping games where entities start out in two groups, take a few seconds after working through the rules to reevaluate them in terms of which entities are in which group. For example, the rules tells us that J and M must be together, and that G needs K. Noting that all four of these people belong to the “tenants” group allows us to formulate a key deduction. • Don’t get sucked into the very common misinterpretation of grouping rules the likes of Rules 3 and 4: “If X is selected, then Y is selected” means that X requires Y; without Y, there can be no X (the contrapositive). But it also means that Y is independent of X — if Y is selected, X could be selected, but doesn’t have to be. This type of rule only goes in one direction.

© K A PL A N

13

LSAT PREP ________________________________________________________________ LSAT Test XI Explained: Section I

The Questions: 12. (D) Another acceptability question — no problem. Rule 3 isn’t violated in any choice, but Rule 4 is violated in (A), which has G without K. (B) splits up J and M, which is forbidden (Rule 5), and (E) contains both M and P (Rule 6). (C) falls to Rule 1, as it contains a group made up of four homeowners and only one tenant. • When facing an “acceptability” question, simply check the choices against each rule, eliminating the ones that violate the rules until you’re left with only one choice. • When checking rules against choices, start with the rules that are easiest to check; in this case, Rules 3 through 6. 13. (C) The second of our Key Deductions comes in very handy here — the J/M pair can’t be selected with G. All four wrong choices pair G with J or M or both, which, as we worked out earlier, simply won’t do. F, J, M is acceptable; all we need to do is add Q (Rule 3) and either R or S to round out the committee. • Don’t be surprised at how easy certain questions at the beginning of games can be, especially when you’ve picked out a few Key Deductions up front. Think of it this way — if you’ve already done the work to answer this question, then getting Question 13 in a few seconds is your reward. • Conversely, if the first few questions in a game are going absolutely nowhere, this is probably the consequence of either missing a Key Deduction, or worse, misinterpreting a rule or a key component of the game’s action. 14. (E) “M is the chairperson” allows us to infer two specific things: J is selected (Rule 5), and M and J must be the only two tenants selected (Rule 2). That means that we’ll need three of the four homeowners to round out the committee. Since M is selected, Rule 6 tells us that P can’t be, which means that Q, R, and S, the three remaining homeowners, will all be chosen. Any of those three can be the answer to the question; the testmakers went with R, choice (E). 15. (B) The same question as 14, except that F is now the chairperson. The first logical step is to find the one other tenant to join F (remember, the chairperson comes from the group that supplies two reps). Neither J nor M will fit the bill; if we take one, we’ll have to take both, giving us three tenants. G’s no good for the same reason — if we take G, we’ll have to select K as well (Rule 4). And that leaves only K as the requisite second tenant. After adding Q to satisfy Rule 3, any two of the remaining homeowners (P, R, and S) could be selected, but any of them could be left off the roster as well, killing choices (C), (D), and (E).

14

© K A PL A N

LSAT PREP ________________________________________________________________ LSAT Test XI Explained: Section I

• Sometimes different questions build on the same concepts. We’ve been able to answer most of the questions so far by focusing on the relationships among the tenants. If you continue to keep this in mind, you’ll undoubtedly benefit from the clear understanding of these relationships in the questions to come. 16. (A) Still concentrating on those tenants, here’s another 10-second question that relies on the issue we’ve seen popping up in all of the previous questions. Basically, the question asks “if F is selected, which of the following people can’t chair the committee?” Deja vu? You bet. While it’s slightly different from Q. 14 and Q. 15, this one’s answered by the same reasoning: If F is selected, G can’t be the chairperson, because G’s selection requires K’s selection, a total of three tenants, which is no good: By now, we know in our sleep that the chairperson comes from the two-person group. Neither J nor M can chair a group with F in it either, for the same reason, which is why neither of those people are listed in the choices (the testmakers aren’t big on Logic Games questions with more than one acceptable answer.) • Different questions often test the same exact concepts. Accordingly, the answers to these questions can be found by using the same reasoning process. Build on your knowledge from one question to the next; remember, questions can be repetitious. So, for that matter, can these bullet-pointed snippets of LSAT wisdom — this one says virtually the same thing as the previous one. (That’s okay, though — Kaplan knows that some points have to be hammered home a number of times before they gain acceptance among students.) 17. (C) The elements of this game should be so familiar to you by now that the chain of deductions for this question should come naturally and quickly, and sound something like this: “No F and no K; without K, we can’t have G. That leaves J and M as the only tenants selected, which means that one of them must be the chairperson.” And that alone is enough to answer the question — J could be the chairperson, and (C) is correct. As for the wrong choices: We saw why G is out (A); since M is in, P must be out (B); the chairperson must be tenant J or M, eliminating (D) and (E). • Don’t be surprised when you encounter easy questions toward the end of a game or a section; they’re scattered around just like the difficult ones.

© K A PL A N

15

LSAT PREP ________________________________________________________________ LSAT Test XI Explained: Section I

18. (A) Interpretation of the hypothetical: choose two homeowners and three tenants. There are numerous ways of doing this, which indicates that we can’t take this analysis much further without additional information. And that’s precisely what we get in each choice — each one is phrased in if-then form. We’ve no choice but to simply try them out. As for (A), if G is selected, must Q be also? Let’s employ the technique of attempting to prove the exception, as discussed in Q. 11 (q.v.). If we select G and don’t have to take Q, then (A) is wrong, so that’s what we’ll try to prove. G needs K, that’s a given. And we need one more tenant to join the committee, in keeping with the question’s requirement that a homeowner chair. Here’s the old familiar issue again — the third tenant can’t be J or M, who come as a pair and would give us four tenants. It must therefore be F, which in turn leads to the selection of Q, from Rule 3. What have we proven? Simply that (A) is a true statement; we tried to create an exception, but we couldn’t make it work. Indeed, if G is selected, then Q must be also. Here are acceptable committees that prove exceptions to the wrong choices: (B) [G K F Q S] (C), (D) [J M K R S] (E) [J M F Q S] • Logic Games answers are objectively correct. As soon as you find an answer, even if it’s choice (A) or (B), have the confidence in your work to circle it and move on without laboring to check the remaining choices. • Questions in which the choices themselves contain if-then statements can turn out to be time-consuming affairs, because there’s often no way to approach them other than working through each choice individually. If the answer happens to be at the bottom of the list, this type of question may take an inordinate amount of time. If you’re strapped for time, questions with if-then answer choices are good candidates to skip and return to later if time permits.

16

© K A PL A N

LSAT PREP ________________________________________________________________ LSAT Test XI Explained: Section I

19. (C) The key to this type of question is in the translation. The right answer will be a pair such that it’s impossible to leave both members off the committee simultaneously; that is, the committee must always include at least one of them. Conversely, the four wrong choices will include a pair of people, both of whom could be left off the committee without violating any rules. The easiest way to get to the answer is to eliminate the wrong choices, by proving that both members of the pair are simultaneously expendable. When you come to the pair where this is impossible, you’ll have the answer. Sound confusing? Here’s how it works: (A) If we can form a group without both F and P, then the committee need not “include at least one representative from this pair.” How’s about [J M K R S]? We’ve seen this committee before; it’s certainly acceptable. So we can get by fine without F and P, thank you, which means we can cross off (A). (B), (D) [F K Q R S] is a valid committee that proves that the members of both of these pairs are simultaneously expendable. (C) Uh-uh, can’t do it. If we disqualify K and Q, then G and F, the people who rely on K and Q (Rules 4 and 3), are out also. With those four out, the other five people must be selected: [J M P R S]. This group, which includes both M and P, violates Rule 6. So the committee must indeed include at least one representative from the K/Q pair. (E) [F G K P Q] is good enough to kill this choice. • If the wording of a question stem is unclear to you, and you’re not sure what the right answer will look like, ask yourself what the wrong choices should look like. Sometimes approaching it from this angle is enough to clarify the meaning of the stem and get you back on track. • Don’t get greedy — if you pick up a lot of easy points from a game, and the wording of the last question throws you, skip it temporarily and come back to it later if time permits. • Use your work from previous questions to help you answer later ones. Some of the acceptable committees used to eliminate choices here are ones we’ve seen before.

© K A PL A N

17

LSAT PREP ________________________________________________________________ LSAT Test XI Explained: Section I

GAME 4 — Apprentices and Projects (Q. 20–24) The Action: An“oddball” game with a matching element: We have to match four apprentices to projects, given an initial assignment and the prospect of two reassignments during the year. The rules will clarify how the reassignments work, but for now there seems to be only one Key Issue: 1) What is the assignment of apprentices to projects after the first and after the second reassignment? The Initial Setup: This game represents the continuation of the trend toward abstraction in Logic Games. It’s very similar to the third game of PrepTest X (the June 1994 exam), “The Word Game” — in both games, there’s not a lot of “drawing” involved, and the “setups” are pretty bare. The most you can do here is jot down the initial assignments, like so:

projects

q

r

s

LOU MAD NOR

t OL

All of your energy in this one will be spent working through the reassignments and thinking about who ends up with which project as a result. Each question will dictate what, if anything, could be jotted down on the page in order to get the correct answer. The Rules: Conveniently, Rules 1, 2, and 3 correspond to Plans 1, 2, and 3, as follows: 1) Plan 1 — Whoever is working on project Q will switch with the person working on S, and the person assigned to R will switch with the person assigned to T. Here’s one way you could represent this rule:

Plan 1

q s

r t

Note that Q and S, which are projects, are not switching places; the apprentices assigned to those projects must switch. 2) Plan 2 — The same basic thing, this time with the apprentice assigned to S and the one assigned to T switching places:

Plan 2

18

s t

© K A PL A N

LSAT PREP ________________________________________________________________ LSAT Test XI Explained: Section I

3) Plan 3 — This one is more specific: No matter what projects they’re working on, Louis will switch with Madelyn. You can differentiate this rule from the previous two by spelling out the first few letters of each name, like so:

Plan 3

LOU MAD

Key Deductions: There’s not enough to go on to form any Key Deductions. You’ll simply have to tackle each question with the help of the rules as stated. The Big Picture: • In somewhat oddball games like this, try to break down the mystique, and approach the games as ordinary everyday situations. Go about it as if you were given this task to perform in real life. • Don’t be intimidated by long and complex-sounding rules; break them down into easy to manage concepts by translating them into your own words. If you’re unsure of a rule’s meaning, form an actual example, in your head or on your page, to help clarify your understanding. • Oddball games tend to involve rules that don’t seem to link up easily, which partly explains our inability to extract Key Deductions from this one. As a result, you can often answer questions in these types of games by focusing on one or two rules at a time; the skill involved is isolating the relevant rule for each question. • If nothing else, this game illustrates what we’ve been telling students for years: Don’t jump in asking “How do I draw it?” Let your sketch — if any — come out of your thinking about the game and its action. • In fact, there was really no need to do any scratchwork on these rules at all. It may have been just as efficient to keep going back to them as written (they’re pretty clear), and just follow their injunctions as needed.

© K A PL A N

19

LSAT PREP ________________________________________________________________ LSAT Test XI Explained: Section I

The Questions: 20. (E) No easing into this game. We need to try out the possible reassignments until we find a way for the second one to result in assigning Nora to project T. Work backwards. Nora has only two ways to get assigned to T — via plan 1, in which case she’d have to be assigned to R and then switch to T; or plan 2, in which case she’s have to be assigned to S (which she initially is) and then switch to T. But there’s no way that the second reassignment could follow plan 1, because there’s no way in the first reassignment to assign Nora to project R to put her in position for a switch to T. Therefore, the only way to assign Nora to T as a result of the second assignment is if that assignment goes according to plan 2. And since Nora has to remain at project S (where she started) after the first reassignment in order for this to happen, the first reassignment must be according to plan 3. Reassigning according to plan 3 and then plan 2 results in the following matchup:

q

r

MAD LOU

s

t

OL

NOR

which corresponds to only choice (E). • The first question can solidify your conception of a game. Answering this one helps to reinforce the action of this somewhat odd game, and reassures us that we have a good grasp of it. 21. (E) The quickest way around this one is to try out the choices. (A) No plan will allow Louis, who begins on project Q, to get to project T after only one reassignment. (B) This would require a plan that allows R’s apprentice to switch with S’s. None exists. (C) would require a Q/T switching plan. (D) No; each of the three plans involves a switch for either L or N in the first reassignment. (E) This one’s possible: If plan 3 were enacted, switching L and M, then N and O would remain where they began after that first reassignment. • Questions in oddball games don’t always supply you with a lot of information, but instead often require you to check each choice individually against the relevant rules.

20

© K A PL A N

LSAT PREP ________________________________________________________________ LSAT Test XI Explained: Section I

22. (A) The test here is two-pronged: The correct choice must contain an assignment that (1) is possible after the first reassignment, and (2) can allow for a reassignment that will place Louis onto project R. (A) meets both criteria: It’s the result of first applying plan 2 to the original lineup, and then plan 3, switching Louis and Madelyn, to get the result we’re looking for — Louis assigned to project R. Choices (B), (C), and (E) aren’t possible following the first reassignment. (D) is possible, but fails on the second criterion: There’s no way in this arrangement to get Louis onto project R from project S. • When looking for situations that could have existed under certain circumstances, search the choices for the one that’s possible. When you find it, there’s no need to continue checking the remaining choices, even if, as in this case, the acceptable assignment or arrangement is found in choice (A). 23. (B) Actually quite simple; on the page, you can quickly list the three possible reassignments that result from each plan, or just work them out in your head. Either way, only one of these assignments can appear in the choices. If plan 1 is enacted on the original lineup, then we switch Louis with Nora and Madelyn with Oliver, and get NOLM assigned to projects QRST, respectively. Using plan 2, we get LMON. Finally, Plan 3, which simply switches Louis and Madelyn, results in MLNO, and that’s what they picked for correct choice (B). • Even oddball games come with ordinary, easy questions. This question (and Q. 21) simply test your understanding of the rules. 24. (A) We just worked out happens when the first reassignment is made according to plan 1; the result is that NOLM get assigned to projects QRST, respectively. No matter which of the other two plans is used for the second reassignment (remember, we’re told that each reassignment is made according to a different plan, so plan 1 can’t be used again), Louis will end up assigned to project T. If plan 2 is used, then the apprentices assigned to projects S and T switch, and if plan 3 is used, then Louis and Madelyn switch. Both result in NOML assigned to QRST, respectively, and lead to choice (A). The remaining four choices clearly don’t jibe with the matchups that will result from the second reassignment. • Read game introductions carefully! Don’t lazily blow off words that don’t seem important. At this point in the game, some test-takers forgot that two reassignments are made during the year, “each according to a different one of the following plans.” Luckily for those test-takers, this fact only really comes into play in this one question, but in other games misreading or entirely missing subtle wording like this can be much more costly. • As mentioned a while ago (Q. 19), the work you put into earlier questions will often help you answer later ones. For example, by the time you get to this question, you’ve already worked out the matchups that result from using plan 1 for the first reassignment.

© K A PL A N

21

SECTION II: LOGICAL REASONING

22

© K A PL A N

LSAT PREP ________________________________________________________________ LSAT Test XI Explained: Section II

1. (E) The hypothesis in question — in both questions! — explains why sea turtles migrate far from their birth sites yet return there up to three decades later: They smell their way back. (E) takes smell out of the migratory equation altogether, rendering the hypothesis highly unlikely. Something else must be guiding the creatures home. (A) If the turtles’ sense of smell were good enough, the “secluded” nature of the birthsite would be no impediment to the turtles finding it again, so (A) is beside the point. (B) If anything this strengthens the hypothesis, by affirming a connection between turtle “preferences” and a physical aspect of their birthsites. (C) So the tags have no effect on turtle behavior? Well, they have no effect on the hypothesis, either. (D) The fact that turtles have a good sense of smell makes the hypothesis more, not less, likely. • Use all clues provided by question stems. Here, the reference to “the hypothesis” identifies what your starting point should be. • Be on the lookout in “weaken” questions for choices that have the opposite effect; that is, choices that actually strengthen the argument.

2. (D) This one follows from much the same logic that drove Q. 1. The hypothesis assumes that the smell of the birthplace establishes itself enough to bring the turtles back. Knowing whether the turtles are in range of the smell before they return is certainly valuable information; if they’re not, then the hypothesis begins to look pretty shaky. (A) Irrelevant. The hypothesis doesn’t involve turtle age. (B) is only partly relevant. Less important than the turtles’ range of migration is whether the birthsite smell can reach that far — which is (D) rather than (B). (C) The possible destruction of turtle birthsites would only be relevant if we knew that it damaged the smell of the sites, but (C) fails to mention that. (E) The issue of gender is irrelevant to the hypothesis. • When a stimulus generates two questions, they usually turn on two very different issues — but not necessarily, especially early in a Logical Reasoning section. Early questions tend to be straightforward. It should have come as no surprise that Qs. 1 and 2 proved so similar! • The right answer to a “relevant information” question will usually point to a key assumption made by the argument’s author.

© K A PL A N

23

LSAT PREP ________________________________________________________________ LSAT Test XI Explained: Section II

3. (C) Paraphrase the reasoning a little: Juanita’s appearance at the zoo, the argument concludes, has to be due to a subway ride, because only the subway or the bus can take her there and the bus is out of commission. That’s (C), in so many words. (A) distorts the reference to “everyone knows.” That common knowledge about the #12 bus doesn’t, in and of itself, lead to the conclusion. Besides, (A) could only be the answer if the author were concluding something about what “everyone knows” — which she isn’t. (B) Exclusivity isn’t really the issue — there’s nothing here about Juanita taking both the bus and subway. (D) But the exception is real: Juanita usually avoids the subway, but today she must have taken it. (E) Huh? What’s “invariable” is that there are only two ways to get to the zoo. What’s “typical” is that Juanita typically avoids the subway. No “substitution” between those two facts is made here. • This is pretty much how the LSAT tests “formal logic” in Logical Reasoning — the stimulus consists of a few formal statements that taken together form a concrete situation. Usually, you’ll be asked to infer or deduce something from the passage; instead, in this case, you’re asked to describe the reasoning in abstract terms. • Method of argument questions in Logical Reasoning are similar to logic questions in Reading Comprehension — both involve determining how or why the author does something. 4. (B) The text presents you with a potentially treacherous obstacle course, but step back from it: Basically, it’s just saying that if the laws regulating “cyberspace” are modeled on either type of regulation (that of phones, or that of PBS), problems will ensue: Neither model is adequate to cover the functions of computer networks. (B) must therefore follow: Contrary to the initial if-clause, network regulation should not be based on previous legislation. Inferably, it should be uniquely crafted to the demands of this new and particular industry. (A) brings in the privacy issue out of nowhere. (C) The scope of the paragraph encompasses the regulation of computer networks, not their genesis. (D) comes from way out in left field. Who should or should not author legislation is no part of the discussion at hand. (E)’s conclusion is not supported. It’s possible that computer networks do much more than mimic the two given systems. There’s no justification in the text for the dismissive charge of “mere duplication.”

24

© K A PL A N

LSAT PREP ________________________________________________________________ LSAT Test XI Explained: Section II

• When a question is worded in the manner of this one (or Q. 7), it means the entire stimulus is evidence only; there’s no conclusion there, but we want something that must be true based on what’s provided. • Don’t assume that the answer to a “conclusion” question requires reading, understanding, and using every piece of evidence! Here, if you were merely to combine the opening if-clause and the last sentence, you could infer the right answer. 5. (E) The conclusion is a policy recommendation: We shouldn’t implement a mandate that youngsters work toward correcting social ills. Why not? Because such mandatory service should only be compelled when the nation’s existence is at stake. Clearly, the assumption is being made that the aforementioned social ills don’t constitute such a threat. If they did, the author wouldn’t object to the plan — it would be justified. (A) The author doesn’t object to the plan on the grounds that its efforts wouldn’t be sufficiently effective. (B) We don’t know what the author would define as a “direct threat” to the nation. (C) Au contraire, if the author believed this she’d be a booster of the plan. This is the opposite of the correct answer. (D) The views of the young are outside the scope. The plan is being evaluated on grounds other than how those affected will react to it. • For assumption questions, use the Denial Test. If (E) were false, then all social ills would be a direct threat to the nation — but that would be a contradiction of the author’s stance. That’s why (E), as written, is a necessary assumption. 6. (E) The proposed ban on cigarette ads stems solely from smoking’s proven health hazards. This line of logic would follow if, as (E) says, only products that are healthy were permitted to be advertised. Look at it this way: If (E) were passed as a federal law, then the logic of the argument would be ipso facto sound. (E) + the evidence would = the conclusion. (A) may sound good at first glance, but the issue of “showing people doing things” kills it. The content of the ads isn’t in question. (A) would be okay if it read “...to advise people to do things that endanger...” But then it’d be (E), wouldn’t it? (B) Nothing is said about the claims made by cigarette manufacturers. The ads aren’t being accused of hypocrisy or disingenuousness. (C), if established, would permit cigarette ads as long as the disclosures were made. But we want a principle that would uphold the proposed ban no matter what.

© K A PL A N

25

LSAT PREP ________________________________________________________________ LSAT Test XI Explained: Section II

(D) is doubly irrelevant, ignoring the main topic (advertising) and bringing in an extraneous one (government standards). • For “principle” questions, keep in mind the tactic mentioned above: Assume that a choice were enacted into law. Would the enforcement of the law support the logic described? • If you chose (A) it’s probably because you read it too quickly, not incisively enough. Don’t let the LSAT’s time demands force you into careless errors.

7. (D) The key here is the word “adult”: All adult male woolly monkeys are larger than all females, and any one will dominate any female. Therefore, if a female dominates any male, then that male cannot be an adult. Note that that’s just the contrapositive of the stimulus claim that “if a male is an adult, then he won’t be dominated by a female.” (A) It’s not inferable that size is the “primary” determinant. There may be other, greater factors at work. (B) Adolescent males are outside the scope. The stimulus fails to mention what must or must not be true about non-adults. (C) Only adult males are within the scope. Contrary to (C), a huge but adolescent male might be bigger than a female and yet not dominate her. (E) All females, adult or otherwise, are covered by the stimulus. Contrary to (E), an adult male will dominate all females, of any age. • Don’t let the brevity of a stimulus cause you to become sloppy. It’s easy to get this one wrong if you fail to focus on that essential term “adult.” • Remember that answers to inference questions must be held to a very high standard of truth. Compare your answer to the stimulus rigorously and carefully before making your final choice. • Note once again that this is the way the current LSAT tests formal logic. Learn to recognize passages that contain such formal statements so that you’ll know when the contrapositive may be of use. 8. (B) If you followed Kaplan advice, you read what T had to say first, and T’s statement only makes sense as a response to a general claim about risk-takers: T implies that people don’t have to fit one label exclusively. S, of course, asserts exactly the opposite view, calling our nation “risk-averse” in general. Both provide examples. (B) delicately speaks to T’s objection: Before you call someone a risk-taker or risk-avoider, consider the context. (A) Both of T’s examples qualify as avoidable risks.

26

© K A PL A N

LSAT PREP ________________________________________________________________ LSAT Test XI Explained: Section II

(C) T makes no reference to what the public believes. (D) T doesn’t respond to S’s reference to “a mathematical ratio,” but to S’s application of one and only one specific label to people in general. (E) T’s comments in no way relate to costs vs. benefits. • When you’re given a dialogue stimulus, read the statement of the second person first. 9. (D) Just because the department head’s authorization is sufficient (enough) to make an announcement important, that doesn’t mean that such authorization is necessary. It’s entirely possible that some, even all, announcements not authorized by the department head are important. (A) How “announcements” are defined is irrelevant. They may take the form of memos, email postings, or standing on a desk and shouting; it doesn’t affect the logic of the argument. (B) The quality that automatically makes an announcement important isn’t “authorization” per se, it’s authorization by the department head. So it doesn’t matter whether someone else has the power to authorize. (C) The key to the argument is in the importance or unimportance of unauthorized announcements. The authorized ones, be there zero or a thousand, don’t affect the conclusion. (E) clouds the issue. This isn’t the time or the place to engage in abstract philosophical discussion about importance. In the context of the argument, “important” is just a condition which may or may not apply to an announcement. We could change the word “important” to “purple” and the logic would stay the same. • Make sure you understand the difference between sufficient and necessary conditions. A sufficient condition is one that in and of itself is enough for a particular result. A necessary condition is one that is required for a result. • Some arguments, though apparently based in real-life situations, are just thinly-veiled exercises in formal logic. Arguments that begin with blanket classifications (“All _____ are ______.”, “Any _____ is _____.”, “No _____ can be _____.”) should catch your attention; the path to the correct answer will require some form of formal reasoning. 10. (D) The “sugar-free” label is literally correct, but many consumers will misinterpret this to mean low-calorie, causing potential harm to would-be dieters. So if it were established that such misinterpretable labeling is unlawful, the conclusion would be entirely justified. (A) and (B) are irrelevant, because the labels described in the argument are literally correct; the “sugar-free” products contain no sugar.

© K A PL A N

27

LSAT PREP ________________________________________________________________ LSAT Test XI Explained: Section II

(C) is too extreme: The labels aren’t written in an overly technical style, so they can be interpreted by the average buyer. The problem is that they’re ripe to be misinterpreted by some consumers from a particular group, namely, those who need to lose weight. (E) is inconsistent with itself, not to mention the argument. If an interpretation of a label doesn’t match the product’s actual properties, it can’t be “equally accurate.” • Always look to eliminate wrong answer choices as quickly as possible. Here, you could have eliminated (A) and (B) after reading only the first line of each choice. 11. (B) The basis for the conclusion is that certain buyers (dieters) will be harmed by the possible misinterpretation of the “sugar-free” label. If, however, it were true that the prohibition of the labels would harm another group of buyers (diabetics), then that would provide strong grounds for challenging the conclusion. (A) would further the author’s interests. Reducing the calories in sugar-free foods would not only eliminate misinterpretation of the labels but result in healthier products. (C) The amount of time it takes for consumers to notice changes in product labels has nothing to do with whether potentially misleading labels should be outlawed. (D) is a mild strengthener. If consumers don’t expect to lose weight right away, it might take them longer to notice that the “sugar-free” products aren’t helping their diet. (E) has no effect on the argument. If the words “sugar-free” are misleading, they are misleading no matter how they happen to appear on the package. • When challenging a recommendation, consider the intent behind the argument (e.g. public health and safety). Look for a choice that suggests the result of the recommended action would be inconsistent with the author’s intention. • Understand the meaning of “should.” When a conclusion asserts that something “should” happen, the mechanics of how it actually would happen aren’t central to the argument. For example, you might believe that there should be world peace. From a logic standpoint, pointing out that world peace would be hard to achieve doesn’t damage in the least the notion that world peace is a worthwhile goal. 12. (E) Ah, formal logic at its most obvious; this stimulus could literally have come right out of a Logic Games section. Deal with the first and third sentences first — they’re more concrete since they contain the word “all.” Combining those, we get an age ordering that looks like this, from oldest to youngest: T > M > D. The sycamores complicate matters; most, but not all are older than the maples. That means that some sycamores must be younger than all the maples, and these young sycamores must therefore be younger than all of the tulips as well. A slightly wordier version of this is choice (E).

28

© K A PL A N

LSAT PREP ________________________________________________________________ LSAT Test XI Explained: Section II

(A) can’t be true. All the dogwoods are younger than the maples, while all the tulip trees are older than the maples. (B) is only possibly true. It’s equally likely that all the dogwoods are younger than all the sycamores. (C) is essentially the same as (B); it could be true, but it doesn’t have to be. (D) Once a tree is older than the oldest maples, we no longer have a benchmark to help us compare ages. So we can’t draw any conclusions about the relative ages of the tulips versus the oldest sycamores. • Note the similarity between the thinking that goes into this question and that utilized for Logic Games. Try to build on the synergy between the sections; skills developed on one section will often come into play on the others as well. • Just as in Games, make sure you understand the difference between statements that could be true and statements that must be true. Just because something is possible doesn’t mean it has to happen. • This argument is almost a miniature sequencing game. Remember that the most useful character in a sequencing game is the one mentioned in the most rules. Here, all the statements in the argument relate to maples, so it was wise to use maples as your benchmark in solving the puzzle. Also, as in Logic Games, work with the most concrete statements first. 13. (A) The argument states that the only effective way to reduce the amount of automobile emissions which plague our atmosphere is to switch to cleaner-burning fuels. But what if people just reduced their amount of driving? Wouldn’t that also reduce the amount of car emissions in the atmosphere? Use the Denial Test on (A): If a reduction in automobile use were more effective than the switch in fuels, then the conclusion that cleaner-burning fuels are “the only effective way” of reducing emissions would fall apart. (B) So what if there were a fuel besides methanol that’s cleaner-burning than diesel fuel and gasoline? This would’t affect the argument. The author doesn’t specifically advocate a switch to methanol; she calls for the use of any cleaner-burning alternative fuels, “such as methanol.” (C) The passage doesn’t suggest offering drivers a choice of fuels, so their preference is immaterial. (D) Not necessary. The existence of other, more serious threats to the environment wouldn’t change the fact that automobile emissions “are threatening the quality of life on our planet.” (E) is beyond the scope. The passage tells us that automobile emissions contaminate both the urban air and the global atmosphere. How the two are related doesn’t affect the argument.

© K A PL A N

29

LSAT PREP ________________________________________________________________ LSAT Test XI Explained: Section II

• To check your answer in an assumption question, try negating it. If you take the assumption to be false, and the argument still works, look for another choice. • Pick up on extreme words like “only,” “always,” “never,” and so on. An argument that concludes a specific practice is the “only way” to accomplish something usually depends on the assumption that all other means are impossible or ineffective. 14. (E) Dr. Libokov argues that land mammals are the cause of the tuatara’s decline, which explains why these reptiles are now found only on islands near New Zealand, which have no land mammals. We’re used to “second speakers” in L.R. disagreeing with the first one, but here Dr. Santos supports this hypothesis, with the additional information that on islands with recently imported land mammals the tuatara population has diminished significantly. (A) Dr. Santos doesn’t find fault with Dr. Libokov’s hypothesis. (B) Dr. Santos goes beyond simply restating Dr. Libokov’s theory. The information on the effect of the recent introduction of land mammals on certain islands is not found in Dr. Libokov’s argument, and therefore serves to provide additional evidence to Libokov’s theory. (C) and (D) Dr. Santos’s information is completely consistent with Dr. Libokov’s argument. • Don’t be intimidated by scientific language. This argument looks long and complicated, but the question itself can be answered fairly quickly. • Use keywords and structural signals to help you understand the argument. The fact that Santos begins with the phrase “in fact . . .” is a strong hint that she’s probably going to supply additional information. • Noticing important words in each answer choice will help you eliminate wrong answer choices quickly and efficiently. Choices (A), (C), and (D) could be eliminated by the words “flaw,” “contradicts,” and “incompatible,” respectively. 15. (D) In preliminary tests, the voice recognition system never gave access to the wrong person. But we don’t know whether the system consistently gave access to the people who were supposed to have it. In other words, it never gave Joe, an unauthorized user, access when he was trying to impersonate John, an authorized user; but that doesn’t mean that John wasn’t denied access as well. From the evidence given, we can’t logically conclude that this system will allow a way of giving access to those people who are entitled; all we know is that it will probably keep out the ones who shouldn’t gain access. (A) The comparison isn’t faulty, because the voiced recognition system doesn’t allow unauthorized users to access the computer.

30

© K A PL A N

LSAT PREP ________________________________________________________________ LSAT Test XI Explained: Section II

(B) While the conclusion is based on a “small initial trial,” the author appropriately qualifies this conclusion with the phrase, “if this result can be repeated in an operational setting.” (C) is beyond the scope. The reasoning applies to the issue of computer security. Other uses of the voice recognition system are irrelevant. (E) Qualifying a conclusion is not a logical flaw. Indeed, the more qualified a conclusion (that is, the more it allows for exceptions), the easier it tends to be to defend. • From reading the question stem, you know there will be a flaw in the reasoning. As you read the argument, be alert for a “leap of faith”: an unwarranted assumption, use of faulty logic, or lack of evidence to support the stated conclusion. • Since outside the scope answer choices deal with other extraneous issues irrelevant to the argument, it’s no surprise that the word “other” will often tip you off to these bogus choices. 16. (C) Combining the last two formal logic statements of this stimulus allows us to infer the conclusion: If low body temperature caused slow reaction, then artificially raising body temperature should cause reaction speed to increase. Since reaction speed doesn’t increase, we can conclude that low body temperature does not cause low reaction speed. (C) just completes that contrapositive. (A) or (B) may be true, but we don’t have enough information to fully justify either. (D) and (E) are specifically contradicted by the data. • The question stem “Which conclusion can properly be drawn?” is logically equivalent to “Which of the following must be true?” It’s not enough to say that a conclusion could be valid. We need a choice that is fully supported by the evidence. • Remember to read the question stem first. If the stem asks you to draw a conclusion, don’t waste time trying to find the conclusion in the argument — it won’t be there. • Not every sentence in Logical Reasoning stimuli carries equal weight. The first two sentences here give background information, primarily so that you’d understand the terms in the final two sentences, but you can derive the answer from the final two sentences alone. 17. (E) The argument draws a conclusion about the high quality of a plan based on inappropriate evidence — the nature of its opponents. So does (E). Chen’s plan “is better for the city” for the same reason that the rail system “will serve suburban areas well”: Those who oppose are self-interested. The anti-Chen bloc has been disposed against the city, while the anti-rail group has been disposed against the suburbs.

© K A PL A N

31

LSAT PREP ________________________________________________________________ LSAT Test XI Explained: Section II

(A), (B) The conclusion of each is a policy recommendation, while that of the stimulus is an evaluation of quality. Not parallel. (C) argues for the superiority of the mayor’s plan mainly by citing its merits; the imputation of bad faith on the part of the opponents is secondary. (D) says that Nomura is a better candidate because of his three qualified supporters. That’s somewhat appropriate evidence, and of course no negative characterization of the opponents is present. • The right answer to a parallel logic question need only be that — parallel; it will rarely be identical in every particular. The important things to consider are (1) the nature of the conclusion and (2) the nature of the relationship between evidence and conclusion. • Questions that ask you to parallel faulty logic have become quite common on the test. Many of the answer choices will sound fairly logical (in terms of their evidence leading to their conclusions), and can therefore be eliminated quickly. 18. (D) The argument criticizes network news on the grounds that the 30 seconds allowed to advocate a point of view is inadequate to fully explain the important facts of an issue. This leads to the conclusion (found in the last sentence) that watching network news increases the tendency to oversimplify issues. But a network news program is more than just that 30 seconds. In order for the conclusion to hold, we must assume that the rest of the news report offers no additional information on the issues presented by the advocates. (A) The argument stops short of recommending that the news programs change their format, so it isn’t necessary that the author assume that viewers want more detailed reports. (B), (C) The passage asserts that network news is oversimplified, but it doesn’t suggest that television news must be simple. Therefore, the author needn’t assume that television is an intrinsically simplified medium. (E) is beyond the scope. The argument addresses only the format of TV and newspaper reports, not the styles or views of the reporters. • Remember the definition of an assumption: anything which the author must be taking for granted as true in order to reach the stated conclusion. • Identifying the conclusion is important for every question in Logical Reasoning, but doubly so for assumption questions, since the conclusion relies so heavily on the author’s assumption. That’s why recognizing keywords like “therefore,” which signals the conclusion in the last sentence, is so crucial.

32

© K A PL A N

LSAT PREP ________________________________________________________________ LSAT Test XI Explained: Section II

19. (A) As opposed to the newspaper, network news, goes the argument, is devoid of the important facts that would allow regular viewers to reflect carefully on the issues, thus increasing their tendency to oversimplify the issues. But if these same viewers also habitually read newspapers, which do contain the important information that TV news lacks, then they don’t necessarily have a tendency to think of the issues in oversimplified terms. Think of it this way: Contrary to the concluding sentence, you may be a network news couch potato, yet not have the tendency to oversimplify public issues, because you also happen to read the newspaper regularly. (B) and (C) strengthen the argument. Both provide further support for the author’s contention that network news doesn’t provide enough information or balanced discussion on public issues. (D) neither weakens nor strengthens the argument. The author’s complaint about TV news isn’t the lack of equal time for opposing views; it’s the lack of sufficient time for thoughtful, balanced discussion. (E) is beyond the scope. The passage deals only with regular watching of network news. Time people spent watching TV in general is irrelevant. • As we’ve seen many times on this test, one way to weaken an argument is to contradict one of its assumptions. Here, the author assumes that regular watching of network news and regular newspaper reading are mutually exclusive activities. Choice (A) contradicts that assumption, seriously undermining the conclusion. • When a stimulus comes with two questions attached, try to answer both of them before moving on. If you don’t, you’ll have to take extra time later to refamiliarize yourself with the argument. 20. (A) The authors of the report concede the critics’ point that the program needs a coherent vision. However, they say, we can’t do anything until we get more funding, and we can’t get the funding until we get our internal problems straightened out. So it’s too early to worry about “the vision thing.” (B) There’s no suggestion that the critics have ulterior motives. (C) The authors “granted that the critics had raised a valid point,” so they must feel that a coherent vision would be appropriate. (D) While they may feel that they know what’s best for the program, there’s nothing here to suggest that the authors think the critics are generally less knowledgeable than they are. (E) The authors don’t deny the desirability of giving the report a single focus; the point is that they’re currently unable to do so.

© K A PL A N

33

LSAT PREP ________________________________________________________________ LSAT Test XI Explained: Section II

• Read critically and carefully. The key to this question was realizing that the authors of the report felt that the critics’ point was valid. That realization bolsters the correct choice and makes it easy to cross off (C) and (E). • Quickly eliminate choices that require you to speculate about motives not touched on in the text. 21. (B) The report’s authors have claimed that they can’t articulate a coherent vision for the program because they must first deal with their internal problems. If it were established that the program’s problems were, in fact, caused by the lack of such a vision, then the authors’ argument wouldn’t hold much water. (A) doesn’t affect the authors’ position. Even if the government doesn’t provide all the program’s funding, it’s still reasonable to believe that continued government funding is necessary for the program to survive. (C) actually strengthens the authors’ claim that no coherent vision can develop or be maintained under current conditions. (D) strengthens the authors’ claim that the program is in danger of losing its funding if it doesn’t overcome its problems. (E) doesn’t help the critics. Even if the program’s reputation for incompetence is undeserved, the program still needs to overcome the perception of incompetence in order to assure continued funding. • When a stimulus includes more than one point of view, be sure you’re clear which side of the argument you’re being asked about. • Underneath the fancy language is an idea you can understand. This question stem is somewhat convoluted, but all you’re really being asked to do is to find an answer choice that weakens the authors’ position. • Instead of being intimidated, you should welcome a point-packed page like this one. You had a chance here to pick up four raw-score points by reading only two arguments. Do this early on, and you’ll save some valuable time which you can use elsewhere in the section. 22. (A) The environmentalist concludes that the oil company has no concern for the environment, charging that the clean-up effort is motivated by the company’s concern for its public image. While the company may be concerned about PR, that doesn’t exclude the possibility that they’re also sincere about their concern for the environment. The assumption being made is that the company can have only one motive for the clean-up. If this is established as true, then the conclusion is properly drawn.

34

© K A PL A N

LSAT PREP ________________________________________________________________ LSAT Test XI Explained: Section II

(B) The survival of the otters is (sad to say) beyond the scope of the environmentalist’s argument. (C) is wrong for two reasons. First, it’s possible that the company is motivated by both profit and concern for the environment. Second, we don’t have any evidence to suggest that the company’s past actions can be used to draw conclusions about its present actions. (D) The point of contention in this argument is the oil company’s motivation for paying for the clean-up. What would have happened if they hadn’t done so is a moot point. (E) is beyond the scope. Previous clean-up projects are irrelevant. • Learn to recognize the various ways in which the testmakers phrase common question types. You’re asked to choose the statement that must be true in order for the conclusion to be properly drawn. In other words, you’re being asked to identify the argument’s assumption, which is something you’ve done a lot of. • Remember that the conclusion doesn’t always come at the end of the argument. Here, the environmentalist’s conclusion is in his first sentence. 23. (A) The first experiment showed that without the parathyroid glands, the level of calcium in rats’ blood dropped severely. This led to the hypothesis that the parathyroid gland raises the calcium level in the blood when it falls below normal. But when they removed both the parathyroid and adrenal glands, the drop in the blood calcium level was much less dramatic. One way to explain this result is if the adrenal glands have the opposite effect as the parathyroid; that is, if they act to lower the level of calcium in the blood. We would then expect the calcium level to be more stable when both the adrenal and parathyroid glands were removed than when just the parathyroid was removed. (B) If the adrenal and parathyroid glands played the same role, we’d expect the calcium levels to decline even more sharply when both were removed, but that’s not what happened. (C) The absence of a parathyroid gland resulted in a steep decline in blood calcium, so (C) is inconsistent with the passage’s evidence. (D) is consistent with the scientists’ hypothesis about the function of the parathyroid gland, but it doesn’t explain the “surprising” results of the second experiment. (E) True or not, this choice fails to explain the “surprising discovery.” • You don’t need any knowledge of biology to answer this question. Underneath the fancy wording is just a situation where event X leads to a big decrease in Z, while events X and Y together still lead to a decrease in Z, just a much less dramatic one. The explanation is that Y has some sort of a tempering effect on X.

© K A PL A N

35

LSAT PREP ________________________________________________________________ LSAT Test XI Explained: Section II

• It may help to visualize the two glands in a way you can understand — the adrenal as a big yellow blob, sucking calcium out of the blood; the parathyroid as a big blue blob pumping it back in again — whatever, you get the idea. • This question really has two parts: you need to find an answer choice that (1) explains the results of the experiment, and (2) is consistent with the scientists’ hypothesis. Instead of making the problem harder, though, it makes it easier, because you have two possible ways of eliminating wrong answer choices. Only correct answer (A) meets both conditions. 24. (A) Mayor Drabble owes Lee a fairly recent political debt, and Lee wants to be appointed to head the arts commission. Since the mayor always repays her political debts “as soon as possible,” she will repay all the previous debts she can before fulfilling her obligation to Lee. So if we want to conclude that Lee will get the appointment he wants, we have to assume that no one else who would like to head the arts commission is ahead of Lee in the line for political repayment by Drabble. (B) The determining factor in the mayor’s repayment of political debts is that debts must be repaid “as soon as possible.” The size of the debt is unimportant. (C) If true, this would be sufficient to ensure Lee’s appointment, but it’s not absolutely necessary. There could be many people to whom Mayor Drabble owes political debts who want to head the arts commission, but as long as Lee’s debt predates theirs, Lee will still get the job. (D) Try the Denial Test: Is it possible that Drabble is concerned about whether Lee is qualified for this appointment? Sure it is. If Lee is qualified, then Drabble’s concern is met and the argument still holds water, which shows that (D) is not a necessary piece of the argument. (E) It’s not absolutely necessary to assume that Lee’s debt can only be repaid in one way. And even if the author does make this assumption, it doesn’t complete the argument. The author still need to assume that the mayor won’t first repay one of her older debts by appointing someone besides Lee to head the arts commission. • To qualify as “an assumption on which the argument depends,” an answer choice must fully bridge the gap between the evidence and the conclusion. • The testmakers always use language literally. “As soon as possible” isn’t just a figure of speech; it’s a specific condition placed on the situation, and it turns out to be the key to solving the question.

36

© K A PL A N

LSAT PREP ________________________________________________________________ LSAT Test XI Explained: Section II

25. (E) (E) is best because it fits the stimulus model pretty closely: Both (E) and the stimulus grant that a particular habit is harmful (smoking/excessive TV watching); both conclude that that habit in moderation isn’t necessarily bad; and both point to analogous evidence: Too much Vitamin A/sleep is bad, but some Vitamin A/sleep is not. (A), (B), (C) The conclusion that second-hand smoke doesn’t necessarily harm nonsmokers is based on evidence about Vitamin A — a different case altogether. That’s called an argument by analogy, and so (A), (B), and (C), which all argue from evidence specific to the conclusion, can be discarded instantly. (D) does draw a conclusion about one topic (a cereal) by way of evidence about another topic (broccoli). But (D) is an argument about personal taste, whereas the stimulus and choice (E) are about benefit vs. harm. Furthermore, (D)’s conclusion is a prediction while the stimulus conclusion is not. Finally, the test sample element of (D) is totally absent from the stimulus. • Attack the parallel logic question by identifying a specific part of the stimulus, and throwing out whatever choices lack that specific part. See above: Only (D) and (E) argue (as the stimulus does) by analogy, so the others can be thrown out right away. 26. (E) The argument concludes that the government should fund the establishment of a supercomputer network on the grounds that (1) no one else has the money to buy all the necessary hardware, and (2) business and universities won’t invest in part of a network without a central coordinating mechanism. But it’s possible that the some business and/or universities could cooperate with each other to coordinate the building of the network themselves, which would obviate the need for government funding. (A) The argument suggests resolving the dilemma by having the government fund the supercomputer network. (B) is beyond the scope. National, international, who cares? (C) The argument makes two main points: we need a supercomputer network, and only the government can pay to set it up. How such a network would be maintained is beyond the scope. (D) is beyond the scope. The argument makes no claims about national preeminence in science. • Learn to recognize choices that are beyond the scope and eliminate them quickly. Here, choices (B), (C), and (D) all speak to issues outside the scope of the argument. Your immediate response to each of them should be “so what?”

© K A PL A N

37

SECTION III: READING COMPREHENSION

38

© K A PL A N

LSAT PREP _______________________________________________________________ LSAT Test XI Explained: Section III

PASSAGE 1 — Martin Luther King (Q. 1–7) Topic and Scope: Martin Luther King’s civil rights philosophy; specifically, the relationship between transcendentalism and King’s philosophy. Purpose and Main Idea: Author re-evaluates the relationship between transcendentalist ideas, especially Thoreau’s, and King’s civil rights philosophy. The passage’s main idea is that past writers have overlooked philosophical differences between Thoreau’s and King’s ideas, as well as similarities between other transcendentalists and King. Paragraph Structure: ¶ 1 hinges on the Keyword “However” (line 9), which signals that the author’s about to offer a new interpretation of the transcendentalist-King connection. The Keywords “first” (line 11) and “second” (line 14) alert you to the author’s specific points — (1) important differences exist between Thoreau’s and King’s philosophies and (2) significant parallels exist between the beliefs of other transcendentalists and King. Predictably, ¶ 2 and 3 take up these points. ¶ 2 contrasts the ideas of Thoreau and King, the Keywords “In contrast” (lines 34-35) serving as a divider between them. ¶ 3 describes a point of commonality between transcendentalists and King — the use of non-violent civil disobedience to combat “unjust” laws. The Big Picture: • This passage is just the sort that you should tackle first. Why? Because topic, scope, and author’s purpose are all revealed early, by line 11. • Be on the lookout for passages that compare and contrast several points of view (as here: Thoreau vs. King vs. other transcendentalists). Such passages always contain questions that hinge on getting the gist of each different viewpoint. The Questions: 1. (D) (D) nicely paraphrases the points made by the author in lines 9-17. These points are then explored in more detail in ¶’s 2 and 3. (A) distorts a detail. Lines 17-20 say that Thoreau’s essay was the only transcendentalist writing with which King was familiar. Moreover, this choice also distorts the main idea of the passage — that King’s philosophy in many ways was more akin to that of transcendentalists other than Thoreau. (B) is a “half right, half wrong” choice. While the author does argue that historians have overemphasized Thoreau’s influence on King, he never claims that King was influenced by other transcendentalists. What he says is that parallels exist between their philosophy and King’s.

© K A PL A N

39

LSAT PREP _______________________________________________________________ LSAT Test XI Explained: Section III

(C)’s first few words sound okay — Thoreau and King did differ on civil disobedience — but the final few hop the tracks. The difference between Thoreau and King wasn’t economic vs. social, but rather individual vs. group and violent vs. non-violent. (E) King was aware only of one transcendentalist tract, Thoreau’s essay “Civil Disobedience.” Thus, he couldn’t have been influenced by Thoreau’s essays or by transcendentalists in general . • Wrong choices in LSAT “globals” are often wrong for very subtle reasons. Read the choices carefully! Don’t endorse a choice simply because the first half sounds right, and watch out for choices that use the passage’s language but misrepresent its ideas. 2. (E) While the author feels that historians have overemphasized Thoreau’s influence on King, he concedes that Thoreau’s essay “Civil Disobedience” did influence King’s philosophy to some degree. (A) and (B) are au contraire choices. (A) is refuted by lines 6-9, (B) by lines 17-20. (C) According to the author, Thoreau endorsed the notion of violent resistance. Thus, the author wouldn’t agree that “Civil Disobedience” provided a model for King’s notion of passive resistance. (D) is beyond the scope. The author discusses only King’s reaction to Thoreau’s essay, not the reaction of transcendentalists in general. • LSAT wrong choices are often written to formula. Classic wrong choice types are those that directly contradict information in the text or go beyond the scope of the text. 3. (B) In the first ¶, the author makes two points: (1) Thoreau has been given too much credit for shaping King’s ideas and (2) this overemphasis has obscured parallels between the ideas of other transcendentalists and King. (B) is an abstract way of saying the same thing. (A) would be correct if the question had asked about ¶ 3, not ¶ 1. (C) is beyond the scope. The passage never even mentions any 20th-century philosophers, let alone their possible impact on King. (D) distorts the thrust of ¶ 1. The author criticizes historians for overemphasizing Thoreau’s impact on King, but he never summarizes their work “on the most important influences” on King. (E) is also beyond the scope. Although transcendentalist philsophers like Thoreau, Emerson, and Fuller are talked about in relation to King’s philosophy, the author doesn’t provide any “background information” on them or other 19th-century transcendentalists.

40

© K A PL A N

LSAT PREP _______________________________________________________________ LSAT Test XI Explained: Section III

• Whenever possible, “pre-phrase” answers to questions. This will direct your search for the correct choice. 4. (E) Lines 14-23 make precisely the same point. (A) is beyond the scope. Thoreau’s ideas are never compared to those of Emerson and Fuller in terms of their correspondence to general transcendentalist ideas. (B) wrongly plays on information pertaining to Thoreau and King. Thoreau was more concerned with individuals, while King was more concerned with society. That’s in lines 26-29. Besides, most transcendentalists were apparently more concerned with individuals than with society. Although the passage doesn’t say so for sure, Emerson and Fuller probably subscribed to this notion. (C) While the passage says that the ideas of Emerson, Fuller, and King were “akin” in some ways, it doesn’t say specifically in what ways. If anything, Emerson and Fuller, given their likely emphasis on the individual, would probably have been no more enthusiastic than Thoreau about mass protests. (D), like (A), is beyond the scope. The passage never discusses whether Emerson and Fuller’s ideas have achieved as much notoriety as those of Thoreau. • Never answer explicit-text questions on a hunch or a vague recollection of the text. Go back and reread the relevant piece(s) of text! The answer to explicit-text questions is always in the passage itself. 5. (D) If you looked up choice (B) in Q. 4, this question should have been a snap. Lines 26-29 explicitly state that King, in contrast to most transcendentalists, was out to reform society instead of the individual. (A) is beyond the scope. That Thoreau condoned violence certainly doesn’t mean that transcendentalists in general approved of such methods. Thus, you can’t conclude that King and transcendentalists in general were opposed on this point. (B) and (E) are similarly beyond the scope. The passage never provides any clues about the views or actions of either transcendentalists in general (though Thoreau was concerned about foreign affairs) or King in the foreign policy sphere. (C) Lines 45-49 make clear that (C) is a point of agreement between King and transcendentalists. • Your work on one question can often help you to choose the correct answer to another question. Work on “paired” questions one after the other.

© K A PL A N

41

LSAT PREP _______________________________________________________________ LSAT Test XI Explained: Section III

6. (C) Boiled down to its essentials, ¶ 1 says that King’s philosophy was in some ways closer to that of other transcendentalists than to that of Thoreau. In other words, Thoreau’s philosophy was not exactly the same as that of other transcendentalists. (A) and (B) are beyond the scope. This passage never discusses either the origins of the civil disobedience doctrine (A) or “contemporary thinkers” (B). It’s restricted to a comparison of the ideas of Thoreau, King, and other transcendentalists. (D) and (E) distort details. Lines 29-32 flatly state that Thoreau wasn’t concerned about changing the government’s foreign policy (D). Lines 26-28 explicitly say that Thoreau was “primarily interested in reform of the individual” (E). • Use the information in the stem to isolate the appropriate part(s) of the text. In this case, since the question’s about Thoreau only, you can safely ignore ¶ 3. 7. (C) The quotes in lines 52-55 come up in the context of a discussion on the link between laws and ethics. As ¶ 3 makes plain, both King and most transcendentalists believed that laws based on ethical principles were just, while those that were not were unjust. (A) is “half right, half wrong.” Yes, the quotes do highlight a similarity between King and other transcendentalists; but there’s nothing in the text to suggest that Thoreau and King differed on the legal vs. moral issue. (B) draws an unwarranted inference. Just because King and transcendentalists agreed on the link between law and morality doesn’t “prove” that King’s philosophy was influenced by the transcendentalists. Indeed, the author makes a point of announcing that Thoreau alone had an impact on King’s philosophy, even though parallels exist between it and the philosophy of other transcendentalists. (D) is beyond the scope. The passage has nothing to say about King’s “philosophy of government,” talking only about when, why, and how King opposed government. (E) is also beyond the scope. The passage doesn’t say anything about how Thoreau decided a particular law was just or unjust. • When a question stem cites line numbers, the answer won’t appear in those lines, but rather in the surrounding lines. Don’t stray too far from the cited context, however. The further away you look, the less likely you are to find the correct answer.

42

© K A PL A N

LSAT PREP _______________________________________________________________ LSAT Test XI Explained: Section III

PASSAGE 2 — Dahl and Democracy (Q. 8–14) Topic and Scope: Dahl’s book Democracy and its Critics; specifically, Dahl’s concepts of democracy and polyarchy and his defense of same. Purpose and Main Idea: Author’s purpose is to define Dahl’s concepts of democracy and polyarchy and to present Dahl’s defense of same. Since this is a descriptive passage, in which Dahl is allowed to speak for himself, the author makes no specific point of his own. The author’s voice, in fact, never even enters the text. Passage Structure: ¶ 1 defines Dahl’s concept of democracy. The last sentence of the ¶ signals a shift in the text’s direction, to the concept of polyarchy. ¶ 2, you can predict from this sentence, is going to define this concept, as well as present Dahl’s arguments in favor of it; and that’s precisely what happens. ¶ 3 switches the text’s direction yet again, by addressing the views of polyarchy’s critics — something you should have expected at some point in the passage (because the first sentence of the text does speak of Dahl’s defense of democracy and polyarchy). Equally predictable is ¶ 4’s focus on Dahl’s response to the critics. This ¶ hinges on the Keyword “Although” (line 49), which indicates that he agrees only in part with the critics. The Big Picture: • This is the sort of passage best left for late in the section. Why? Because the author’s purpose is not made clear in ¶ 1. (Indeed, you have to read the entire passage before you can figure out what the purpose is.) Instead, you get a lot of details, which don’t allow you to make any guesses as to the direction in which the text is headed. Moreover, a quick scan of the question stems should suggest that this is not a particularly easy question set. • Authors usually have a point of view on a given topic, but occasionally they don’t. Sometimes they merely report objectively on a topic. The questions attached to such a passage will want to know whether you’ve noticed the absence of the author’s “personal voice.” The Questions: 8. (A) Just prior to describing polyarchies as “centrifugal,” the author talks about how they are based on a “diffusion of power away from a single center....” (B) focuses on a detail from the wrong ¶ — ¶ 3 — where polyarchy is not defined, but criticized. (C) Lines 25-27 indicate that political parties don’t acquire “concentrated power” in polyarchies because of the vote, a point that even critics of the system seem to agree with (in lines 38-42).

© K A PL A N

43

LSAT PREP _______________________________________________________________ LSAT Test XI Explained: Section III

(D) focuses on (and distorts) a detail from the wrong ¶ —¶ 4. (E) gets the ¶ right, but the detail wrong. • Vocabulary questions don’t come up very often on the LSAT. But when they do, context is all important, with the correct answer generally appearing in the lines immediately before the chosen word. 9. (E) The author’s purpose, as has already been pointed out, is to present Dahl’s defense of polyarchy. This defense wouldn’t be completely intelligible unless the author also made some reference to critics of the system. You could also have gotten the answer by process of elimination. The verbs “refute” (A), “advocate” (B), and “suggest” (C) all imply that the author himself has something to say on the issue. But that’s not the case. He simply relates the views of Dahl and his critics. Even (D) — though it begins with the more neutral-sounding “point out” — implies that the author’s got an opinion. (D) also wrongly connects details that have nothing to do with each other. • Logic questions ask how or why the author has done something — organize a ¶, include a detail, cite an opinion, etc. Whenever you’re faced with this type of question, be sure to keep the author’s purpose in mind. 10. (B) A neat, boiled-down paraphrase of lines 27-32. (A) Au contraire, lines 29-30 explicitly state that parties “do not ask what the majority thinks of an issue....” (C) Au contraire. Lines 32-34 flatly say that pressure groups are organized by their membership. (D) Au contraire aussi. Lines 17-19 mention that elected officials “are accountable to a broad-based electorate,” not political parties. (E) While lines 17-19 relate that officials have to answer to the electorate, there’s nothing to indicate that the function of political parties is to see that they do so. • In explicit-text questions that don’t provide line numbers, use the information in the stem to relocate the pertinent part(s) of the text. (Political parties are discussed only in ¶ 2.) Also, watch out for au contraire choices, a classic wrong answer type.

44

© K A PL A N

LSAT PREP _______________________________________________________________ LSAT Test XI Explained: Section III

11. (A) The last two sentences of ¶ 1 say that Dahl defends polyarchy because it “approximates” the democratic ideal. Put another way, Dahl doesn’t believe that polyarchy — or, for that matter, any other system of government — can ever fully live up to the democratic ideal. For Dahl, polyarchy’s the next best thing. (B) contradicts Dahl’s beliefs. According to lines 7-9, Dahl thinks that hierarchy is an inevitable part of any governmental system, polyarchy included. (C) also contradicts Dahl. Lines 9-11 make it clear that Dahl also thinks that equal access to resources is an unreachable goal in any governmental system, once again polyarchy included. (D) is beyond the scope. The passage never compares Dahl’s beliefs about polyarchy vs. other political systems on the issue of political party growth. (E) goes against Dahl’s beliefs in the same way as (B) and (C). Lines 55-63 clearly show that Dahl believes a connection exists between polyarchy and economic interests. • In inference questions, watch out for choices that use the passage’s language but misrepresent its ideas. 12. (B) The passage’s point about polyarchy vs. democarcy is that polyarchy, the real, can never completely live up to democracy, the ideal. (Note that the correct answer to the previous question is a strong clue about the correct answer to this question.) The only scenario that makes this real/ideal distinction is (B) — the musical score, the ideal, can never be reached because of the limitations of instruments, the real. • Application questions are rare on the LSAT. But when they do appear, your task is to pick the hypothetical scenario that is parallel or analogous to the scenario outlined in the text.

© K A PL A N

45

LSAT PREP _______________________________________________________________ LSAT Test XI Explained: Section III

13. (E) The basic criticism (in ¶3) of Dahl’s model of polyarchy is that traditionally wealthier and better organized groups in society have dominated the political system at the expense of smaller groups. Thus, if it could be shown that these smaller groups actually had as much influence on policy as their wealthier, better organized counterparts (E), Dahl’s model would be strengthened. (A) and (B) reflect the criticism leveled against polyarchy by its detractors. (C) and (D), though not specific criticisms leveled by detractors, clearly conflict with Dahl’s concept of polyarchy. Therefore, if true, they would not strengthen his defense. • Strengthening an argument in Reading Comp. is much the same as doing so in Logical Reasoning. Look for choices that are consistent with the argument in question, and watch out for choices that are either inconsistent or beyond the scope. 14. (D) The only choice that encompasses the correct topic, scope, and purpose of the text. (A) dwells on an issue that is taken up only in ¶ 4. (B), (C), and (E) wrongly blow up details — in ¶’s 2, 4, and 2, respectively — into the passage’s major theme. • Descriptive passages are just as likely as argumentative ones to have “global” questions attached to them. You should recognize that this question is nothing more than a “primary purpose” question with a reworded stem.

46

© K A PL A N

LSAT PREP _______________________________________________________________ LSAT Test XI Explained: Section III

PASSAGE 3 — Species Diversity in the Amazon Basin (Q. 15–21) Topic and Scope: Species diversity in the Amazon River basin; specifically, two contrasting hypotheses about what type of climate conditions contributed to the diversity. Purpose and Main Idea: Author wants to highlight evidence supporting a new view that climatic disturbance — not climatic stability, as stressed by the old hypothesis — has been the driving force behind the rich variety of species in the Amazon basin. Paragraph Structure: ¶ 1, introductory in purpose, describes the old belief (Sanders’ hypothesis) that species diversity resulted from a stable climate. Note the Keyword “however” at the beginning of ¶ 2: For the next 3 ¶s, the direction of the argument changes. The focus now is on the new belief that it was climatic instability that led to the diversity. More specifically, ¶ 2 describes a puzzling observation about bird species that inspired the new view. ¶ 3 describes Haffer’s explanation of (his new hypothesis about) that puzzle; his idea is that differing and changing climates in upland and lowland areas could have served to isolate animal populations and lead to species diversity. The basic idea in ¶ 4 is that Haffer’s hypothesis, though unproven, has been influential — it has prompted new research that adds support to his view. The Big Picture: • Like most science passages, this one is thick with details. The key ideas here (two contrasting hypotheses) shouldn’t be hard to see, but the amount of detail makes things tougher to navigate. A savvy test-taker might have chosen to move on to the next passage and save this one for last. • Note the simple, “classic” structure of this passage: The author starts by citing the old, outdated view (¶ 1), and then moves to a lengthier discussion of the new view (¶s 2-4). We’ve seen this “old view/new view” format time and again in LSAT reading passages; being aware of it makes reading a tougher argument (forming a roadmap of the key ideas) that much easier. And having the roadmap under your belt, of course, scores points! • As you read a tougher science passage, remember that most questions (including Inference and Explicit Text questions) will typically hinge on key points. In other words, you’re rewarded for not getting bogged down in the fuzzy details. A strategic focus on the key ideas — the big idea and paragraph topics — always gains the most points. • While pushing your way through a tough passage, don’t stop to dope out details. Here, for instance, you could have moved confidently into ¶ 3 without being 100% clear on the ‘bird puzzle’ in ¶ 2. You can always go back to specific details and understand them in greater depth, as needed.

© K A PL A N

47

LSAT PREP _______________________________________________________________ LSAT Test XI Explained: Section III

15. (C) According to ¶ 1, Sanders observed a link between species diversity in the deep sea and stable conditions there, and then concluded that a similar stability could explain species diversity in the Amazon. Lines 14-17 are explicit: “Sanders argued that the Amazon tropical rain forest is analogous to the deep sea: because the rain forest has a stable climate, extinction should be rare.” (A) is tricky, but the problem is that it refers to facts: It’s the cause of the diversity that’s in question, not the diversity itself. The stem asks for an assumption, not a fact. (B) distorts the passage: It refers to a detail at the end of ¶ 1 relating to rain forest trees, not to life in the deep sea. (D) A “dependable supply of water” is never mentioned anywhere in the passage. (E) The passage never mentions an “equivalent” rate of speciation in the Amazon and the deep sea. • Let’s do a tally; let’s count the questions in this set that refer to key points. This one’s the first. (C) paraphrases important info in ¶ 1, the basic idea in Sanders’ theory. 16. (C) (C) is a literal paraphrase of ¶ 4: The author makes clear that Haffer’s hypothesis is unproven but very influential. (A) The phrase “intriguing and complete” implies that Haffer’s hypothesis is proven. Not so. (B) doesn’t work because the author never suggests that Haffer’s hypothesis is only “partially correct.” (D) is tricky, but the low rate of extinction isn’t really a point of difference between Sanders and Haffer. According to ¶ 3, Haffer says that the uplands became ice age refuges that protected existing species from extinction. The implication is that the uplands were islands of climate stability, and that’s consistent with Sanders’ idea that where climate is stable, “extinction should be rare” (lines 16-17). So the author probably wouldn’t say that Haffer says anything different or better than Sanders on the extinction issue. (E) On the contrary: Lines 45-47 state that “Haffer’s hypothesis appears to explain the distribution of species as well as the unusual species diversity.” • Here’s another question that dovetails with your roadmap of key paragraph ideas. That makes it an easier point. • Notice how much filler there is in these choices. Zero in on important words and don’t linger on the rest. (A)’s “intriguing and complete” — no way! Bag it! (B)’s “partially correct”? No way! Bag it! And so on.

48

© K A PL A N

LSAT PREP _______________________________________________________________ LSAT Test XI Explained: Section III

• Don’t waste time with confusing choices like (D). You’re in charge, and (C) is clearly the right answer. Bag it and move on. 17. (D) The second sentence of ¶ 3 states that “Amazon lowlands are drier than uplands...” (D) just puts a little spin on that. (A) ¶ 3 states that the lowlands were a “near-desert arid plain” during the ice ages, not at present. (B) is inconsistent with the same point: The ice ages caused the lowlands to become desertlike. (C) contradicts the last sentence of ¶ 3: The lowlands were the site of “the development of new species as existing species on the lowlands adapted to changing climates.” (E) Population density in the upland/lowland areas is never discussed. • Use your clues! In the stem, the words “uplands” and “lowlands” key you to ¶ 3, and the relevant point comes near the beginning. • Expect right answers to put a slight paraphrase spin on the text. 18. (A) The author cites an old hypothesis (¶ 1), identifies its weaknesses (¶ 2), and then highlights a new and much stronger one (¶s 3-4). (B) can be eliminated fast — no “recently observed phenomenon” is mentioned anywhere in the opening paragraph. (C) Only two hypotheses are mentioned, Sanders’ (the single “bad” one) and Haffer’s. (D) is even further from the mark. The passage discusses two hypotheses, not just one. (E) The first half of (E) is sort of okay, though it lacks the precision of (A). But the second half doesn’t fit at all. The passage never mentions experiments of any type. • Another question that harkens back to the roadmap — the string of key paragraph ideas. And with this passage, as well, we have a time-worn, “classic” argument format, which makes answering a logic question like this one much, much easier.

© K A PL A N

49

LSAT PREP _______________________________________________________________ LSAT Test XI Explained: Section III

19. (E) The detail cited in the stem is mentioned parenthetically, in the middle of a sentence describing the repeatedly changing climatic conditions (13 ice ages) that “facilitated the development of new species.” (E) paraphrases that idea. (A) The relevant sentence is explaining species diversity, not species distribution — a distinction that the author (and Haffer) want to make clear. (B) The previous sentence states that ice age refuges served to protect species from extinction, but the author mentions the number of ice ages for a different reason. (C) The passage never suggests that “most existing species” have survived “periodic climatic disturbances in the Amazon basin.” (D) Only one type of climatic disturbance is cited in ¶ 3: ice ages. • Read the immediately surrounding context for answers to questions like this one. 20. (E) (E) spells out the major weakness of Sanders’ theory, and jibes with key info in ¶ 2: Doubts were raised about Sanders’ theory because of puzzling distribution patterns of bird species in the basin. (A) The passage never says that the basin actually experienced winter and glaciation. Haffer proposes only that climate conditions in the basin changed during the ice ages. (B) If anything, Sanders failed to see the differences between the lowlands and uplands. (C) The passage never cites a “relatively high rate of extinction” during the ice ages. ¶ 3 explains that upland conditions during the ice ages aided species survival. (D) Sanders’ estimates of length of time of species survival are never an issue. • Yet another answer that jibes with the roadmap.

50

© K A PL A N

LSAT PREP _______________________________________________________________ LSAT Test XI Explained: Section III

21. (E) (E) is consistent with Haffer’s hypothesis as it’s described in ¶ 3. Haffer proposes that during the ice ages the lowlands saw significant climate changes: They became desert-like, while the uplands were lush refuges. Those changes, Haffer theorizes, would explain species diversity and distribution. If fossilized pollen from desert-loving plants was found in the lowlands, that would jibe with (and thus support) Haffer’s explanation. (A) isn’t specific enough. And if the rising lake levels occurred in the lowlands, that would weaken Haffer’s theory. (B) would definitely weaken Haffer’s theory: He argues that the uplands became lush refuges, hospitable to existing rain forest species. (C) would also tend to weaken Haffer, who argues that the lowlands became very dry. “Insignificant” reductions of monsoon rains would not result in desert-like conditions. (D) Yet another weakener: Haffer believes that during the last ice age the uplands were lush, not arid. • “Strengthener/Weakener” questions like this one can be unnerving, but staying focused and cool pays off: In this case the answer corresponds to the now-familiar point about the lowlands being dry during the ice ages. • Here’s the tally: by our count, a full 5 or 6 points jibe with roadmap-level points (# 15, 16, maybe 17, 18, 19, and 20). So don’t give up on tough science passages. They conform to the same structure patterns as other passages, and there are always easier points to add to your score.

© K A PL A N

51

LSAT PREP _______________________________________________________________ LSAT Test XI Explained: Section III

PASSAGE 4 — Women Medical Practitioners (Q. 22–27) Topic and Scope: Women medicos in medieval Europe; specifically, the extent to which historians’ usual research methods have underestimated the extent to which women practiced medicine in the Middle Ages. Purpose and Main Idea: Author assesses previous methods and findings, and argues that historians should “broaden...the scope” (line 42) of their research in order to improve our “understanding of women medical practitioners’ role in medieval society” (lines 66-67). Paragraph Structure: ¶ 1 identifies a problem in most histories of medieval medicine: Most historians “continue to equate the term ‘woman medical practitioner’ with ‘midwife’” (lines 5-8), and thus underestimate the number of women involved in other medical specialties. In ¶ 2 the author asserts that historians have to go much further by asking why, in general, there is so little evidence about women practitioners; the ¶ ends with the suggestion that when the definition of practitioner is broadened out, we get more accurate data and conclusions about women. In ¶ 3, the author approvingly cites a study by Pelling and Webster that illustrates the value of research that moves beyond a focus on “offically recognized and licensed practitioners” (lines 45-46). Pelling and Webster found evidence for a much larger number of women practitioners in London in 1560. ¶ 4 cites other potential research paths (beyond purely statistical studies) that would add to our knowledge of women practitioners. The Big Picture: • As with the science passage, this passage’s argument fits a time-worn pattern on the LSAT: An older view or approach is flawed and should be replaced with an updated one. [The thesis here is that the traditional methodology (which confines women to the “midwife” category and stresses the “learned” — heavily male — medical categories) is flawed and there’s now a new approach that can remedy such shortcomings.] The point is that while the topics of passages change on each test, the basic argument patterns recur. These “classic” passage arguments makes a strategic reading easier. • In a passage that cites different studies (here, Wickersheimer’s and Jacquart’s; Pelling’s and Webster’s; and Gottfried’s) it’s worth circling names for easy reference. You can bet that questions will ask about them. 22. (B) (B) covers two important bases: first, the author’s criticism of the prevailing methodology that restricts knowledge of women medicos; and second, the idea that historians should broaden the scope of their research. (A) distorts the passage. Nowhere is it implied that women medicos were more prevalent in England. Pelling’s and Webster’s study just happened to focus on London.

52

© K A PL A N

LSAT PREP _______________________________________________________________ LSAT Test XI Explained: Section III

(C) is inconsistent with ¶ 2: In lines 26-41 the author acknowledges that there are limitations in the historical record, but goes on to stress that historians’ methodology contributes to the lack of info about women. (D) misses the point in the last ¶: There the author urges that future studies go beyond statistics to encompass social issues that could enhance our understanding of women medicos. Contrary to (D), those studies haven’t yet occurred. (E) accurately summarizes the point in the last ¶, but that’s not the main idea. The thesis is that historians should change their approach. • With choices as long and wordy as these, speed up. Skim the choices impatiently, looking for buzzwords or phrases — like “methods of study” at the end of correct choice (B) — that connect with the passage’s focus. Don’t waste time with a choice that doesn’t “click” as you skim it — move on to the next one. 23. (B) The error referred to in the stem is that historians have equated a broad group (women medical practitioners) with a single category from that group (midwives). The same error is found in (B): the equation of science with a single branch of science, biology. None of the other choices moves in the same way from a general group to one example from that group. 24. (E) Gottfried’s study, in contrast to Pelling’s and Webster’s, identified only a very small number of women medicos in England. The implication is that it’s a study whose scope is too narrow. (A) We know that P & W’s study went as far as 1560 (line 52). Is that “late”? Who knows? In any case there’s no sense that if Gottfried studied the same data as P & W he’d come up with better conclusions. (B) As the author stresses in ¶ 2, inaccurate sources are only part of the problem. At least as significant is historians’ (such as Gottfried’s) narrow methodology. (C) Nothing is ever said about any “loss” of medieval documents. (D) makes an unwarranted connection between Gottfried and the idea in the last ¶ that historians should extend their studies to broader social analysis. Gottfried is merely compared (unfavorably) to Pelling and Webster. • Here’s a question that’s easily referenced if you circle names as you read.

© K A PL A N

53

LSAT PREP _______________________________________________________________ LSAT Test XI Explained: Section III

25. (B) (B) fits squarely with the author’s advice in the last ¶: Medical historians should make use of the work of other historians to better understand the social conditions affecting women medicos. (A) is way too broad; it ignores women medicos altogether. (C) misses the point in the last ¶. The various types of specialties are of interest, but ¶s 1-3 suggest that much of this work is done. (D) This work, too, is done: At least some historians, including the author, already know why “women medical practitioners” are equated with “midwives.” (E) This choice is tempting — at first it seems to fit nicely with the idea in the last ¶ — but a second glance reveals that it, like choice (A), overlooks the specific issue of women medicos. • If you key your reading to the gist of each ¶, it’s hard to miss a point like this one. 26. (D) Wickersheimer and Jacquart come up early, where the author discusses equating women medicos with midwives. W and J’s study showed that midwives were a minority of women medicos in France. (A) W and J’s study focused on women medicos and midwives, not on medical specialties in general. (B) Tempting, but it seems to mix up W and J with Pelling and Webster. The former pair studied the categories (midwives vs. women medicos), not the extent to which women medicos in general were undercounted. (C) Midwives were already recognized. They were over-recognized. W and J sought to correct that distortion. (E) Same problem as (C): W and J weren’t primarily focused on midwives. • Another question that’s easily referenced. If time was so short that you couldn’t even read the last passage, you could still take a quick stab at a question like this.

54

© K A PL A N

LSAT PREP _______________________________________________________________ LSAT Test XI Explained: Section III

27. (D) The author’s primary purpose, as we’ve noted, is to urge historians to extend the “parameters” of their research so as to improve knowledge “about the variety and prevalence of women’s medical practice” (lines 58-59). (A) is tricky, but it suggests that new methodologies are already in place. The author cites two statistical studies that have moved beyond the older methods, but then goes on to assert that the bulk of the work is undone. (B) is too imprecise. The passage isn’t really about “revising definitions,” and “concepts” is stretching things too. It’s about women medicos and better methods of studying them. (C) Too vague. The passage doesn’t focus on different “analyses” as much as different methods. (C) is also too neutral in tone. Our author argues forcefully for a new methodology. (E) “Chronicling” is definitely the wrong verb. And the author never really discusses historical developments. The focus is on methodology, not actual historical events. • When things boil down to two choices — as they may have boiled down to (A) and (D) — don’t lose your cool and guess: You may end up with the second-best choice. Instead, look for very concrete differences between the two choices. For example, the verb “arguing” in (D) is much better than “describing” in (A). It matches the author’s very opinionated tone of voice. That single item identifies (D) as the better choice. • Note, too, that (D) is a paraphrase of the answer to Q. 22, the earlier global question. The idea in (D) is not new.

© K A PL A N

55

SECTION IV: LOGICAL REASONING

56

© K A PL A N

LSAT PREP _______________________________________________________________ LSAT Test XI Explained: Section IV

1. (D) We can resolve the paradox that Megatrash loses more and more money in lawsuits, yet its profits increase, by finding another, favorable influence on the company’s bottom line, which would offset their losses in court. A decrease in competition could reasonably be expected to accomplish this. (A) is beyond the scope of the argument. We’re not concerned with why Megatrash is vulnerable to lawsuits, only with the surprising results in spite of the success of such lawsuits. (B) would, if anything, compound the paradox (assuming that these contributions happened to coincide with the timing of the lawsuits), because it suggests another way in which Megatrash would be giving away its money. (C) adds to the paradox, because it suggests legal injunctions against Megatrash that would only harm the company’s profitability. (E) suggests that Megatrash loses money even when it wins a lawsuit, which doesn’t help us to explain the company’s surprising success. • In paradox questions, first isolate the basic contradiction, and try to paraphrase it in simple terms; e.g. Megatrash loses money, yet makes money. • Immediately reject choices that have no bearing on the actual paradoxical result. For instance, the paradox here centers around making money, and (A) has no direct bearing on that. On the other hand, be wary of choices like (B) and (C) that deal with the correct issue, but add to rather than resolve the discrepancy. 2. (C) Morris’s complaint is that history can be used to justify points of view on either side of an argument, which is intended to dispute Lewis’s contention that the lessons of history can be used to avoid repeating mistakes. Morris is doubtful that history can be used in such a way; to him, history’s lessons are equivocal. (A) Morris doesn’t dispute the existence of “uncontested historical facts.” He challenges the use of these facts to prove a point. (B) Actually, Morris suggests that history’s “inexhaustible storehouse of events” is too broad to be useful in the context of Lewis’s argument. (D) Morris doesn’t care how “past mistakes” are defined; but rather points out the impracticality of using history to learn from them. (E) is beyond the scope. The issue here is not whether the past affects specific events in the present; it’s whether people living in the present can learn from the past.

© K A PL A N

57

LSAT PREP _______________________________________________________________ LSAT Test XI Explained: Section IV

• Always read the question stem first. In this case, the stem tells you that you should focus on Morris’s view, and also alerts you to the fact that Morris is in fundamental disagreement with some part of Lewis’s argument. • In dialogue questions, even when the question stem doesn’t tip you off to focus on the second argument, it’s smart to read the second person’s statement first. Often, you’ll be able to infer a great deal about the first person’s position just by the second person’s response. 3. (E) Did the scientists prove that food irradiation is safe, as supporters of food irradiation would have you believe? No; they just haven’t yet found a way in which it’s unsafe. The problem with this type of reasoning is that until the safety of food irradiation is positively proven, there remains the possibility that it’s unsafe in some as-yet-undiscovered way. The “claim” referred to in (E) is that irradiating food is safe. The supporters do use a lack of contradictory evidence (evidence that would show it to be unsafe) as conclusive evidence for the claim. (A) raises a side issue. While the scientists’ motives may have affected their testing, the bottom line is that their research, biased or not, did not positively affirm the safety of food irradiation. (B) Sure they are. We’d expect that the supporters of food irradiation would want to prove that it’s safe. But we’re concerned with their logic, not their motivations. (C) The question addresses food irradiation only with respect to safety issues. Other arguments for or against the practice are beyond the scope. (D) It’s not the responsibility of the food irradiation supporters to recount the scientists’ research in detail. In any case, since the scientists’ results were negative (they failed to prove that food irradiation is unsafe), the specific data from their research has no bearing on the logical structure of the argument. • Flaw questions are the LSAT’s way of asking “What’s wrong with this picture?” So visualize: Ask yourself, if someone used this argument on me, would I buy it? Why not? • The correct answer to a flaw question will strike at the heart of the argument. Avoid choices that nitpick at side issues. • Negative proof is no proof at all. Consider: “No teacher, I can’t prove that the dog ate my homework. But you can’t prove he didn’t, so there.”

58

© K A PL A N

LSAT PREP _______________________________________________________________ LSAT Test XI Explained: Section IV

4. (D) In order to yield their full nutritional value, lima beans must be neither overcooked nor undercooked. Since different size beans cook at different speeds, the best way to insure that all the lima beans are cooked “just right” is to cook together those beans that are close to the same size. (D) neatly brings together the three otherwise unrelated topics in the paragraph — size, cooking time, and nourishment. (A) Just because undercooked lima beans aren’t completely digestible does not mean that lima beans that are completely digestible are overcooked. (A) tries to get you to make this erroneous connection and therefore conclude that completely digestible lima beans will suffer the fate of overcooked ones — namely, a loss of nutrients. This ignores the possibility that digestible lima beans could be perfectly cooked, complete with the full array of nutrients. (B) asks you to make an unwarranted connection between the lost nutrients of overcooked beans and the nutrients the body can’t assimilate from undercooked beans. Even if certain nutrients in undercooked beans fail to be digested by the body, there’s nothing to indicate that these are the same nutrients that are lost from overcooking. (C) The key phrase here is “fully cooked.” There’s nothing to suggest that fully cooked lima beans of any size are anything less than fully digestible. (E) We don’t have enough information to draw this conclusion. Which has more value — many poorly digested nutrients or a few fully digested ones? • A stem that begins “if the statements above are true...” is asking you to either draw a valid inference based on the evidence presented, or recognize the conclusion that the author is leading to. • When a stimulus speaks of two different groups or categories, be on the lookout for wrong choices that rely on unwarranted connections between them. • When an answer choice is based around a specific judgment, as in (E), literally ask yourself: “Does the author provide enough information in order for this judgment to be properly drawn?” 5. (C) This argument contains an assumption so reasonable that we almost don’t notice it: namely, that the only people who will read the proposed article in Homeowners’ Journal are people who have already decided to do their own renovations. (C) undermines this assumption by asserting that the availability of information on home renovation encourages certain people (those who would have otherwise hired professionals) to do their own work. The author’s stated concern is homeowners’ safety, but if (C) is true, then the proposed article will result in more homeowners exposing themselves to the risk of lead poisoning than would be the case without the article. (A) neither weakens nor strengthens the recommendation. The passage deals only with homeowners who have lead-painted houses and are planning renovations.

© K A PL A N

59

LSAT PREP _______________________________________________________________ LSAT Test XI Explained: Section IV

(B) if anything slightly strengthens the recommendation, by providing more evidence that some homeowners will choose not to hire professionals to do their renovation. (D) also strengthens the recommendation. If qualified professionals are unavailable to many homeowners, then more of them will go the do-it-yourself route, and there will be a greater need for safety information. (E) is beyond the scope. The author is concerned specifically with homeowners who “believe they could do [their renovations] less expensively themselves.” • When asked for a statement that argues against an author’s recommendation, look for a choice that, if true, would conflict with the author’s stated goals. • Be on the lookout for answer choices that do the opposite of what you’re looking for — as, for instance, choices that strengthen the argument when you’re asked to weaken it, or vice versa. 6. (B) Applied to the passage, principle (B) restates the author’s argument: Some homeowners are going to do their own renovations no matter what anybody else says, and Homeowners’ Journal has a responsibility to provide them with safety information. (A) supports the recommendation made in the second sentence of the paragraph (that professional contractors should do renovations involving lead-based paint), but has no bearing on the recommendation about an article. (C) weakens the recommendation. If it were established that homeowners’ magazines shouldn’t provide their readers with information about risky procedures, then Homeowners’ Journal shouldn’t run any articles about renovations involving lead-based paint. (D) We don’t have any information to suggest that renovations involving lead-based paint are dangerous to anyone but the people actually in the house at the time. (E) If we were to establish that the lack of the proposed article constitutes “discouraging,” then this principle might serve as justification for the author’s position. However, this seems like quite a stretch. • In some principle questions, you’ll be asked to find the principle that justifies the author’s position, while in others you’ll be asked to find the one that goes against the author’s argument or recommendation. In either case, the correct choice will be an abstract restatement of the position you’re asked to take. • The answer to principle questions is usually the choice that covers the key points of the argument. • Try not to skip passages that apply to two questions — or if you do skip ‘em at first, be sure to get back to ‘em by section’s end. You can often get two answers in less time than it takes to handle two “regular” questions.

60

© K A PL A N

LSAT PREP _______________________________________________________________ LSAT Test XI Explained: Section IV

7. (C) Science, the author tells us, has deprived us of the assurance that people are important. The conclusion: science “has taken away much that is... greatly valued.” Since we aren’t told that science has taken away anything else, the necessary assumption is that the assurance of humans’ importance is “greatly valued.” (A) The author may or may not believe this to be true. However, it’s not necessary to consider religion superior to science in order to complete the argument. (B) The author states that science has “eased our everyday life,” which, at least in some ways, suggests the opposite of (B). Either way, (B) has no bearing on the argument. Whether or not people have resisted the advance of science, science has advanced. (D) Similar to (A). It’s possible, but not necessary, that the author thinks the world was better in the pre-scientific age. (E) contradicts the passage. If the passage assumed that science and technology satisfied people’s need to feel important, then just what “highly valued” thing did science take away? • An assumption is any statement that is necessary to complete the argument; it’s something which the author must take for granted to be true in order for the conclusion to follow logically from the evidence. • To find the central assumption, look for a gap between the terms of the evidence and the terms of the conclusion. Then find the choice that bridges that gap. • Looking for an assumption is not the same thing as looking for a statement that the author agrees with or believes in. Whether or not the author subscribes to the judgments in (A) and (D) is irrelevant; the bottom line is that a valid assumption must be the necessary missing link in the argument. 8. (C) The author doesn’t deny that too much smoking causes health problems. The point of his argument is that those problems are not the legal or moral responsibility of the tobacco industry. To support this point, the author reasons by analogy that too, much candy also causes health problems, and this, too, is public, undisputed information; yet no one looks to sue the candy makers. (A) The author does seem to believe in open acknowledgment of the hazards of smoking, but this is only a springboard into the main thrust of the argument. (B), (D) The references to the candy industry are evidence, present only to support the author’s main point, that the tobacco companies are blameless.

© K A PL A N

61

LSAT PREP _______________________________________________________________ LSAT Test XI Explained: Section IV

(E) turns the argument inside out. The purpose of the argument is to convince the reader that the tobacco companies shouldn’t be held responsible for the health problems of their customers. Why not? Because the candymakers aren’t. Why worry about a Never-Never Land where candy companies are held responsible for cavities? • A question stem that asks for the “main point” of the argument is simpy asking for the argument’s conclusion. • Remember, the conclusion isn’t always at the beginning or end of the passage. Here, it’s in the middle. • Recognize when an issue or idea is brought up specifically to support a larger overriding point. Choices that focus too heavily on a side issue, such as (B), (D), and (E) here, will never be the answer to a main point or conclusion question. 9. (E) The argument attempts to compare the legal and moral responsibility of the tobacco industry (which many consider to be harmful) with that of the candy industry (which most consider benign). The argument is therefore most vulnerable in the areas in which the two industries are most dissimilar. Choice (E) strikes at this vulnerability by noting the rather glaring difference between cavities and lung cancer. (A) True, a connection between candy-eating and tooth decay isn’t clearly established, but the author hasn’t really attempted to make one, nor does the argument depend on one being made. Remember, the point of the argument isn’t to vilify the candy industry, but to defend the tobacco companies. (B) No, the argument doesn’t assume that candy is the cause of all cavities. It asserts simply and reasonably that excessive candy eating can cause tooth decay. (C) It’s not necessary for the argument to define “excessive” and “long-term.” Both are well-understood from the argument’s context to mean “too much.” (D) picks up on the least significant point in the passage. The beliefs of “many people,” alluded to in the second sentence, are mentioned mainly as a contrast to the author’s position, and play no part in the logical chain of the argument. • In a question that asks you to critique the argument, look for a problem with the central thrust of the passage. Don’t blame the author for not embellishing or fully documenting minor supporting points — it’s generally not the author’s responsibility to do so. • Read actively and critically! The candy analogy should have sounded pretty weak upon first reading, and so recognizing choice (E) as the correct choice here should have been a cinch.

62

© K A PL A N

LSAT PREP _______________________________________________________________ LSAT Test XI Explained: Section IV

10. (D) Rather than considering the effects of veterinary treatment on the entire population of injured seabirds, Jonathan attacks Lydia’s proposal on the specific grounds that it would be inhumane to prolong the lives of “the most seriously injured birds,” which are in fact the birds that her proposal serves least well. (A) Don’t read too much into the argument. There’s nothing here to suggest that Jonathan is anything but sincere when he states his admiration for Lydia’s concern. (B) Jonathan doesn’t imply that Lydia is motivated by self-interest. He simply points out what he considers to be a flaw in her proposal. (C) Don’t get hung up on the phrase “as all wildlife should.” Jonathan limits his discussion to the most seriously injured birds. He has no stated problem with aiding birds with lesser injuries. (E) Jonathan doesn’t claim that birds are not worthy of sympathy. In fact, he expresses admiration for Lydia’s feelings. • When you see a stimulus in dialogue form, let the question stem guide your focus. You still need to read both sides of the argument, but don’t invest a lot of time in the half of the stimulus that’s not directly covered by the question. • Elimination can be a particularly useful strategy on technique questions, because wrong answers are usually easy to recognize. • In dialogue questions, recognize when one person’s argument contains a personal attack against the other person. Since Jonathan doesn’t attack Lydia in this way (he expresses admiration for her feelings and attacks a point in her proposal) three of the five choices can be eliminated quickly. 11. (B) Since trees release carbon dioxide when they decompose, the official suggests that harvesting old trees before they die will help reduce the levels of carbon dioxide in the atmosphere. The assumption being made is that old trees that are harvested for manufacture won’t decompose after being harvested. Choice (B) directly contradicts this assumption, seriously weakening the argument. (A) is beyond the scope. The effect of logging on animal species is irrelevant to the passage. (C) neither weakens nor strengthens the argument. The reason for making room for young trees is that they absorb more carbon dioxide than old trees. How much carbon dioxide they contain has no bearing on the official’s logic. (D) is consistent with the official’s argument. Harvesting old trees would probably reduce the amount of “wood and other organic debris found on the forest floor.”

© K A PL A N

63

LSAT PREP _______________________________________________________________ LSAT Test XI Explained: Section IV

(E) If anything, this choice strengthens the official’s argument. Since young trees absorb more carbon dioxide than old trees, the longer it takes for newly planted trees to reach maturity, the more carbon dioxide they’ll absorb from the atmosphere. • In weaken the argument questions, look to see whether there’s a key assumption that connects the passage’s evidence to its conclusion. Contradicting this assumption will undermine the author’s reasoning. • If no answer choice offers a contradiction of the central assumption, look for a choice that offers a reasonable, plausible, alternative explanation of the situation. • Read answer choices carefully. Notice how the alteration of one word (“absorb” vs. “contain”) can invalidate a possible answer choice. 12. (D) The data: old people who play bridge tend to have better short-term memory than old people who don’t play bridge. The original conclusion: Playing bridge helps older people’s memory. The author counters this with a perfectly reasonable alternate explanation: Bridge is more fun for those who already have good short term memory, and that explains the survey’s results just as readily. (A) The representativeness of the sample is not questioned. (B) doesn’t go far enough. The author does concede the relationship between bridgeplaying and short-term memory, but does more than question the conclusion: She offers an alternative explanation. (C) The people’s reasons for playing bridge are irrelevant to both the original conclusion and the alternate explanation. Having good short-term memory can’t be considered a “motive.” (E) The passage doesn’t critique the reasoning behind the original conclusion; it simply offers a reasonable alternate explanation. • Know the difference between “correlation” and “causation.” The survey found a correlation between playing bridge and having a good short-term memory, but that’s not enough to conclude that playing bridge causes one to have a good short-term memory. • As we’ve said over and over, reading each question stem first can save you a lot of time and trouble. In a technique question, for instance, the stem tells you to focus more on the nature of the argument than on its content.

64

© K A PL A N

LSAT PREP _______________________________________________________________ LSAT Test XI Explained: Section IV

13. (A) Preventive treatment is cheaper than treatment of the actual disease, it says there. But how much would it cost to conduct widespread genetic screening? What if detecting people with rare genetic flaws that are amenable to preventative treatment is so infrequent that the money saved in detecting these cases is actually less than the money it costs for widespread screening? For the author’s point to be valid, the combined cost of widespread screening and the preventative treatment for the flaws detected by that screening would have to be cheaper than simply treating diseases as they appear. (B) While the idea that most genetic flaw-linked diseases are preventable lends support to the argument, it’s not absolutely necessary to support the conclusion. Even if we’re only talking about one preventable disease, it may still be cheaper to prevent it than to treat it. (C) is completely inconsistent with the argument. The author’s point is that widespread screening would allow more preventive treatment and therefore reduce the amount of resources needed to treat people who develop disease. (D) We’re specifically told that “a person with a rare genetic flaw that is detected can receive the appropriate preventive treatment.” How does it benefit the argument to assume that the health-care system is inadequate or incompetent? (E) is not an assumption, but a variation on the conclusion: that widespread screening and preventative treatment of patients with rare genetic flaws will save the health-care system money, which inferably could be put to other uses. • Once again: An assumption is anything that must be taken for granted as true in order for the conclusion to follow logically from the evidence. To check your answer, try assuming that the statement you have chosen is false (The Denial Test); if the argument still works, seek another choice. • Arguments that suggest a new route to financial success often hinge on the assumption that the cost of implementing the new scheme will be lower than the amount saved or generated compared to the current practice. (As in real life, it’s a good idea to scrutinize this assumption closely.) 14. (A) The prevailing theory of paranoia in both decades mentioned corresponds to the dominant images of paranoia in the films of those decades. In the absence of any counterevidence, it is reasonable to conclude that this correlation is true for all periods. (B) The passage doesn’t speculate about the root causes of paranoia. (C) The evidence states that the roots of paranoia shifted from ungrounded fears in the 60’s to real societal threats in the 70’s. However, that’s not the same as saying that the world itself is actually becoming more dangerous or threatening; there’s simply been a shift in the way social scientists perceive the causes of paranoia. Furthermore, (C) entirely ignores the central issue of the ways in which films reflect the paranoia theme.

© K A PL A N

65

LSAT PREP _______________________________________________________________ LSAT Test XI Explained: Section IV

(D) Paranoia itself doesn’t change. The change described is in the way in which paranoia is viewed by social scientists and filmmakers. (E) The passage provides no evidence about the curability or incurability of paranoia in any era. • When you’re asked to draw a conclusion, look for a choice that takes into account most or all of the evidence presented in the passage. Choices that omit crucial aspects of the passage will usually be wrong. • Conversely, stay away from choices that seem to depend on hidden assumptions, or that deal with subjects not directly discussed. 15. (E) To prevent harm to the plants, the fungicide must be diluted to ten parts water to one part fungicide. But is this concentration of fungicide strong enough to eliminate powdery mildew from rose plants? If the author wants to conclude that safely diluted fungicide is both safe and effective for this purpose, she must assume the answer to be yes. (A) Who cares about other methods? What we want to know is whether this fungicide is safe and effective. (B) The passage doesn’t claim that the fungicide is harmless to the animal kingdom. (C) raises yet another side issue. The possibility of other fungal infections doesn’t change the fungicide’s effectiveness in eliminating powdery mildew. (D) is irrelevant, because we are specifically told that the fungicide in question has the capacity to completely eliminate powdery mildew. Whether the effectiveness of all fungicides in this respect is based on eliminating the mildew completely has no bearing on the argument. • Don’t read too much into conclusions. Here, the passage doesn’t argue that anyone or everyone should use the fungicide. The author claims only that the fungicide is a safe and effective way to rid rose plants of powdery mildew. • The more Logical Reasoning questions you do, the faster you’ll be able to spot and eliminate outside-the-scope answer choices. Instinctively, you should be able to read these choices and immediately think to yourself “who cares?” 16. (A) Recycling plastic invariably produces plastic of a lower grade than the original plastic. Plastic of lower quality than the lowest commercial grade is unusable. Therefore, recycling plastic of the lowest commercial grade will result in an unusable product. (B) We’re specifically told that products made from recycled glass can be equal in quality to normally made glass products, so there’s no reason to conclude otherwise.

66

© K A PL A N

LSAT PREP _______________________________________________________________ LSAT Test XI Explained: Section IV

(C) Nothing in the passage deals with the relative costs of recycled glass products or quartz sand glass; we therefore can’t draw any conclusions on the subject. (D) Who knows whether materials scientists can tell recycled plastic from virgin plastic? Where did these materials scientists come from anyway? The passage doesn’t address the visual differences between recycled and virgin plastic, so there’s no support for this conclusion. (E) The passage offers no information on a comparison of the quality differences between various grades of glass relative to various grades of plastic. • Not all of the information in every stimulus will lead directly to the right choice. Here, for example, the first sentence turned out to be unimportant to the correct answer. However, every part of the stimulus is there for a reason. You needed to understand what the first sentence didn’t say, in order to eliminate choices (B), (C), and (E). 17. (B) The teacher warns journalists who use unattributed quotes of the possible damage to their reputation: Quotes that don’t have the characteristics of a good anecdote won’t get published. The student’s demurral (that journalists might as well make up juicy quotes and not bother with sources at all) assumes that such phony quotes can and will get published. But if the teacher takes it for granted that only actual quotes are publishable — if, in other words, phony quotes are outside her scope — then the student’s response is wholly inappropriate. (A) The issue of marginal vs. primary journalistic activity is made up by (A) and has no connection to this debate. (C) What the student has confused is not the characteristics of statements and their context, but the intent of the teacher’s statement in the first place, which was to point out a danger in a particular journalistic practice. (D) The student makes an extreme statement, but does not take the teacher’s position to extremes, as (D) alleges. You can tell that he doesn’t commit (D)’s error because he departs from the teacher’s terms to bring in the fakery issue. (D) would demand that he stay within her terms and take them to an absurd conclusion. (E)’s “three criteria” must be plausibility, originality, and interest; but both the student and the teacher link those words with “or” rather than “and.” A quote that meets any one of those characteristics qualifies as a “good anecdote.” Neither party makes any reference to the three traits being jointly met. • The LSAT always tests your understanding of the difference between sufficiency and necessity — sometimes subtly, as here. Note that the teacher complains that juicy interest is necessary if an unattributed quote is to be published, while the student assumes that merely making up juicy quotes will be sufficient for publication. It’s a distinction worth keeping in mind.

© K A PL A N

67

LSAT PREP _______________________________________________________________ LSAT Test XI Explained: Section IV

• We can’t remind you often enough to pay special attention to “keywords” — structural signals — as you begin to attack an argument. Simply noticing the phrase “This is so because” tells you that the conclusion was just stated and that the evidence is to follow. The result is less work, but more accurate work, for you. 18. (A) If, on first reading of the teacher’s statement, you were unclear about the link between a journalist’s reputation and his or her use of unattributed quotes, (A) provides it: If (A) is true, if the journalist’s rep is damaged when a boring unattributed quote is “spiked” (as they say in the newspaper game), then that rep is at the mercy of the “logic of anecdote,” just as the teacher alleges. (B) implies that unattributed quotes are in the minority. But they still could risk damaging a reputation. (C) contrasts plausibility with originality (which is no help, since the teacher deals with both separately and equally), and raises an issue (the reliability of the journalist) that’s not part of the scope. (D) The reason for the lack of attribution is irrelevant. (E) Knowing that journalists with secret sources are prized by their publishers adds nothing to the logic. If anything, it goes against the idea that unattributed quotes present risk to a writer’s reputation. • When looking over the choices in a “strengthen” question, be sure to look for a statement that somehow bolsters an assumption that the author is making, and be sure to reject choices that stray beyond the argument’s scope. 19. (A) The key word in this choice is “intrinsically.” The argument dismisses the value of clinical trials for new surgical procedures on the grounds that the effectiveness of such a procedure is “transparently related to the skills of the surgeon who uses it.” Even so, it is reasonable to expect that some surgical procedures may prove to have harmful effects in and of themselves, regardless of the surgeon’s ability. (B) Yes, it’s theoretically possible that the challenged proposal is deliberately crude. But the argument is specifically a response to the proposal itself, whether it be a first draft or a final product. That’s not a flaw. (C) The importance attached to the “skills of the surgeon” doesn’t imply that the passage assumes that a surgeon’s skills remain constant. The argument would work just as well if it read “the skills of the surgeon at the precise moment in his career that he happens to be performing the operation in question.” (D) There’s clearly a qualitative difference between administering drugs and performing surgery. Specific scientific evidence proving the dissimilarity is unnecessary.

68

© K A PL A N

LSAT PREP _______________________________________________________________ LSAT Test XI Explained: Section IV

(E) is way off. Failure to acknowledge the “good faith” behind a proposal may be impolite, but it isn’t a logical flaw. • In flaw questions, it can be useful to keep in mind the limitations of the test format. Every argument has to be put forth in a single paragraph. So it’s unreasonable to demand detailed documentation in support of every piece of evidence. 20. (D) Let’s break down the argument using variables. If X (most apartments have ants), then Y (management calls the exterminator). But Not X (most apartments are ant-free), so Not Y (no exterminator). The same structure is repeated in choice (D): If X (the band will play the picnic), then Y (most employees will attend). But Not X (the band will be out of the country), so Not Y (most employees won’t attend). (A) If X, then Y. X, so Y. (B) Not X, unless Y. Not Y, so Not X. (C) X and Y must happen before Z. Y has happened, so Z will happen soon. This is obviously flawed, but in a different way than the original. (E) Either X or Y must happen before Z. X will not happen so Z will not happen. Like (C), this contains a logical flaw, but one that’s not parallel to the flaw in the stimulus. • In parallel reasoning questions, be concerned primarily with the structure of the argument. Try not to get caught up in specifics; it’s the general framework of the original that you want to find in one of the answer choices. • Some parallel logic questions lend themselves well to algebraic representation. If you can reduce the structure of the argument to letters, as we’ve done above, do so. Then look for the choice with the exact same structural breakdown. • As a shortcut, notice that only choices (A) and (D) are based on if-then statements. Since the original argument is based on an if-then statement, you could have eliminated (B), (C), and (E) right away. 21. (E) Technology can compensate for the loss of many natural resources by developing methods that depend on other, more widely available resources. But what about clean air and clean water? They are certainly important. If technology can’t figure out a way for us to breathe something else or drink something else, then, contrary to the final concluding sentence, we certainly can run out of important natural resources. (A) The argument allows for the continued demand for old resources, such as shipbuilding timber.

© K A PL A N

69

LSAT PREP _______________________________________________________________ LSAT Test XI Explained: Section IV

(B) is consistent with the argument. Technology doesn’t replace the lost resources; it shifts the demand for them elsewhere. (C) and (D) are beyond the scope. Though the supply and demand terminology may sound like economics, the passage makes no claims regarding the actual costs of using old resources or developing new technologies. • In “weakener” questions, look for the argument’s unspoken assumption. Here, the assumption is that technology is capable of finding alternatives to all natural resources. By contradicting the assumption, you undermine the conclusion. • Be suspicious of choices such as (A) and (B) above, that attack the author’s illustrative examples. These are rarely crucial to the argument’s foundation. 22. (B) The stimulus goes like this: Two events (visits to Paulsville and Longtown) are mutually exclusive. One event is asserted — the Paulsville visit — followed by a possible exception (“unless Salisbury is visited”) that is immediately eliminated (“Salisbury is out of the question”). This leaves the Paulsville visit confirmed, which means (the 1st sentence again) that Longtown is out. (B) fits this model exactly, though it presents the information in a different order: Tom’s support for Parker and for Chung is mutually exclusive; it’s asserted he will support Parker, with a possible exception (Mendoza’s not applying). This exception is rendered inoperative (Mendoza won’t apply); Tom’ll therefore support Parker, so he won’t support Chung. (A) comes real close. It fails because it doesn’t raise and then immediately remove a possible exception. The 2nd and 3rd sentences combined mean that the chef won’t use radishes, but we can’t properly deduce, as (A) does, that she won’t use peppers either. She still could. (C)’s 2nd and 3rd sentences are contrapositives of each other, while the 1st sentence has no relation to the logic of the other two. This is far from a schematic of the stimulus. (D) The “either one but not both” appears nowhere in the stimulus. Nor does the prospect of a third alternative (i.e. pest-resistant crops). (E) presents two things that are mutually exclusive in terms of cost, but mutually dependent in terms of utility. • The right answer to a parallel logic question has to feature the same kind of evidence, leading to the same kind of conclusion, as the stimulus — but those elements don’t have to be mentioned in the same order. The way they’ve rearranged the parts of choice (B) is typical. The altered order doesn’t affect the way the argument works.

70

© K A PL A N

LSAT PREP _______________________________________________________________ LSAT Test XI Explained: Section IV

23. (B) Did you start by sorting out the three symptoms that are part of the “invariant” and “complex” syndrome affecting migraine sufferers? We learn that they are: anxiety in childhood; migraines in adolescence; and depression from age 20. Fine. But the stimulus ends by concluding that the childhood anxiety leads to the migraines and depression, and we simply cannot infer this cause and effect. As (B) reminds us, maybe all three symptoms have an as-yet-unmentioned cause. It’s eminently possible. (A) The author is examining only those adults, however many there are, who suffer from migraines. The argument doesn’t rely on those adults making up any particular proportion of the general population. (C) “Inconsistent” means that two things cannot both be true, but that doesn’t apply here. The conclusion isn’t inconsistent with the evidence; it simply doesn’t follow from it. (D) The representativeness of those studied is irrelevant, because the author’s conclusion doesn’t claim to move from a sample to the broader population. (E) The reasoning isn’t rendered dubious because children and adolescents were left out of it. What’s dubious is the causality asserted. • When arguments assert cause and effect, be alert! The testmakers want to test your ability to discern proper causal reasoning from improper, and just as in Q. 12, you need to be able to tell the difference between correlation and causation.

© K A PL A N

71

LSAT PREP _______________________________________________________________ LSAT Test XI Explained: Section IV

24. (A) The conclusion tries to discredit the mainstream economic theory that says that manufacturers do what they do simply on the basis of consumer demand. The flaw in this theory, according to the author, is that manufacturers create, or at least shape, demand: Just watch television! Those theorists do, so they’re being disingenuous. The claim in question, then, is an important piece of evidence en route to proving that mainstream theorists have some sort of hidden motive. (B) No. The conclusion is clearly everything that follows the final comma of the stimulus. How can you can tell? Because if you begin with that clause, the question “How so?” is raised. “Economic theorists have a different motive than scientific truth? How so? Why do you say that?” The rest of the stimulus answers that question; hence it’s the evidence. (C) No. What’s argued against is the implicit claim that theorists advance this particular theory solely in the name of scientific truth. (D) No. Any “objection to the conclusion” would have to deal with the conclusion’s subject: economic theorists, not manufacturers. (E) The statement about manufacturers, as explained above, is an essential step in the logic; it’s not just background filler. • It’s appropriate to end this document with a reminder about one of the fundamentals: To be sure you’ve located an argument’s conclusion, make sure that the statement implicitly asks “How so? Where’s your evidence?” The rest of the stimulus should provide that evidence. Remember, if you don’t identify the conclusion properly, everything else is up for grabs. Analyze carefully!

72

© K A PL A N

I.N. LL3106 Rev.A Printed in the USA